M

Chiudi

Espansione di Taylor: teoria

Teoria Espansione di Taylor

Home » Espansione di Taylor: teoria

Espansione di Taylor: teoria

Presentiamo la nostra dispensa relativa all’approssimazione di Taylor, uno strumento che consente di approssimare una funzione f con un polinomio e a meno di un resto, asintoticamente per x \to x_0. Tale approssimazione è costituita da un polinomio di grado pari all’ordine di derivabilità di f in x_0, risultando quindi più precisa per funzioni maggiormente derivabili.

Questa proprietà possiede numerose importanti applicazioni, che spaziano dagli efficaci utilizzi nel calcolo dei limiti alla classificazione di un punto stazionario di una funzione in base al primo ordine di derivata non nullo in tale punto.

In questo articolo esploriamo tale importante strumento trattando le seguenti domande:

  • In cosa consiste la formula di Taylor e cosa sono i resti di Peano, Lagrange e Cauchy?
  • Cosa sono le funzioni analitiche?
  • Come si calcolano gli sviluppi di funzioni elementari come l’esponenziale, logaritmi, funzioni trigonometriche e serie geometriche?
  • Quali applicazioni al calcolo dei limiti e alla classificazione dei punti stazionari possiedono le serie di Taylor?

Mostreremo inoltre alcuni esempi particolari di applicazioni, come una dimostrazione dell’irrazionalità del numero di Nepero e il calcolo di valori approssimati di numeri trascendenti.

Ogni argomento è corredato da esempi e intuizioni, oltre che da esercizi svolti. Il testo consente dunque di avere una rapida introduzione al tema degli sviluppi di Taylor, fino a fornire approfondimenti dettagliati e curiosità difficilmente reperibili altrove.

Consigliamo la lettura dei seguenti articoli sulla teoria collegata:

Segnaliamo inoltre le seguenti raccolte di esercizi:

Buona lettura!

 

Scarica la teoria

Ottieni la dispensa teorica sullo sviluppo in serie di Taylor.

 

Autori e revisori

 

Sommario

Leggi...

Questa voce si occupa dell’importante argomento dell’approssimazione di funzioni sufficientemente regolari mediante polinomi, focalizzandosi sull’intorno di un punto specifico. Tale tipo di approssimazione è comunemente nota come espansione di Taylor. Dopo aver introdotto le principali definizioni, la trattativa si concentra sullo studio delle espressioni del resto nelle forme di Peano, di Lagrange e di Cauchy.

Successivamente, vengono presentati i più rilevanti sviluppi delle funzioni elementari. Infine, la voce fornisce diverse applicazioni della formula di Taylor, inclusi i criteri per la determinazione degli estremi relativi, la risoluzione delle forme indeterminate nel calcolo dei limiti di funzioni e la soluzione di problemi classici di approssimazione di numeri trascendenti.


 
 

Introduzione

Introduzione.

Consideriamo una funzione di variabile reale f: (a,b) \to \mathbb{R} che sia derivabile nell’intervallo (a,b). Dato un qualsiasi x_0 \in (a,b), dalla definizione di derivabilità si ha:

    \[ f'(x_0)=\lim_{x \to x_0} \frac{f(x) - f(x_0)}{x-x_0}. \]

In maniera equivalente, possiamo scrivere che

(1)   \begin{equation*} f(x)=f(x_0)+f'(x_0)(x-x_0)+R_1(x;x_0), \end{equation*}

dove R_1(x;x_0) è un termine di resto che soddisfa la seguente identità:

(2)   \begin{equation*} \lim_{x \to x_0}\frac{R_1(x;x_0)}{x-x_0}=0. \end{equation*}

L’uguaglianza (1) indica che, in un intorno di x_0, la retta passante per il punto P_0=(x_0,f(x_0)) di coefficiente angolare f'(x_0) è tangente alla curva f(x). Inoltre, nello stesso intorno fornisce una buona approssimazione della f a meno di un errore di ordine superiore al termine lineare.

    \[\quad\]

    \[\quad\]

Figura 1: sviluppo di Taylor al primo ordine, ovvero (1).

    \[\quad\]

    \[\quad\]

Inoltre, è facile verificare che la retta tangente alla curva è l’unica retta passante per il punto P_0=(x_0, f(x_0)) il cui resto associato verifichi la proprietà (2). Infatti, data una qualsiasi retta passante in P_0 di coefficiente angolare m \in \mathbb{R}, si ha:

    \[\lim_{x \to x_0}\frac{f(x)-[f(x_0)+m(x-x_0)]}{x-x_0}=f'(x_0)-m,\]

e questo resto è nullo se e solo se m=f'(x_0). Supponiamo ora che la funzione f sia derivabile due volte nell’intervallo (a,b). Allora applicando De L’Hôpital due volte otteniamo:

    \[\begin{aligned} 	\lim_{h \to 0} \frac{f(x+h) - 2f(x) + f(x-h)}{h^2} & \stackrel{\text{\tiny De L'Hôpital}}{=} \lim_{h \to 0} \frac{f'(x+h) - f'(x-h)}{2h} 	\\ & \stackrel{\text{\tiny De L'Hôpital}}{=} \lim_{h \to 0} \frac{f''(x+h) + f''(x-h)}{2}= f''(x)  \end{aligned}\]

Pertanto, possiamo trovare un’approssimazione più precisa di f in un intorno di x_0 utilizzando un polinomio di secondo grado. Dato c \in \mathbb{R}, si aggiunge un termine quadratico alla retta tangente precedente, ovvero:

    \[y=f(x_0)+f'(x_0)(x-x_0)+c(x-x_0)^2.\]

Analogamente a (2), introduciamo il resto associato a questo polinomio di secondo grado come

    \[R_2(x; x_0) :=  f(x)-[f(x_0)+f'(x_0)(x-x_0)+c(x-x_0)^2],\]

e chiediamo che sia verificata la seguente condizione al limite:

(3)   \begin{equation*} \lim_{x \to x_0}\frac{R_2(x;x_0)}{(x-x_0)^2}=0. \end{equation*}

Sostituendo R_2(x;x_0) con la sua definizione, otteniamo

    \[\lim_{x \to x_0}\frac{R_2(x;x_0)}{(x-x_0)^2}=\lim_{x \to x_0}\frac{f(x)-[f(x_0)+f'(x_0)(x-x_0)]}{(x-x_0)^2}-c=\frac{f''(x_0)}{2}-c,\]

dove abbiamo utilizzato il teorema di de l’Hôpital per ottenere il rapporto incrementale della derivata:

    \[\lim_{x \to x_0}\frac{f(x)-[f(x_0)+f'(x_0)(x-x_0)]}{(x-x_0)^2}=\lim_{x \to x_0} \frac{f'(x)-f'(x_0)}{2(x-x_0)}=\frac{f''(x_0)}{2}.\]

Dunque la condizione (3) è verificata se e solo se il parametro c soddisfa l’uguaglianza

    \[ c = \frac{f''(x_0)}{2}. \]

Notiamo dalla figura seguente come l’approssimazione intorno al punto sia migliorata.

    \[\quad\]

    \[\quad\]

Figura 2: sviluppo di Taylor al secondo ordine..

    \[\quad\]

    \[\quad\]

È possibile generalizzare queste considerazioni e approssimare funzioni k volte differenziabili in (a,b) con polinomi di grado minore di o uguale a k: tale risultato porta il nome di teorema di Taylor, ed è uno dei pilastri della teoria del calcolo infinitesimale.


Notazione di Landau.

Nel seguito faremo ampio uso della notazione di Landau, perciò ne richiamiamo in questa sezione la definizione e le proprietà principali.

Definizione 1 (notazione di Landau). Sia x_0 \in (a,b) e siano f,g:(a,b) \to \mathbb{R} due funzioni tali che

    \[ \lim_{x \to x_0} f(x)=\lim_{x \to x_0} g(x)=0. \]

Diciamo che f(x) è un infinitesimo di ordine superiore a g(x) per x \to x_0, se

    \[\displaystyle \lim_{x \to x_0} \frac{f(x)}{g(x)}=0. \]

In tal caso utilizzeremo la corrispondente notazione di Landau, ovvero scriveremo che f(x)=o(g(x)) per x \to x_0, e leggeremo che ‘‘f(x) è un o-piccolo di g(x)’’.

    \[\quad\]

Vediamo ora alcune proprietà fondamentali degli o-piccolo. La dimostrazione è un’immediata conseguenza della definizione che il lettore può utilizzare per fare pratica, perciò qui ci limitiamo ad enunciarle:

    \[\quad\]

Algebra degli o-piccolo (inteso per x \to x_0, nel caso generale):

  • Se c \in \mathbb R è una costante non nulla, allora

        \[ 			 c\cdot o(f(x))=o(c \cdot f(x))=o(f(x)). 			\]

  •  

  • Se f(x) = o(g(x)), allora per ogni \alpha > 0 si ha

        \[ 			[f(x)]^\alpha = o( [g(x)]^\alpha ). 			\]

  •  

  • Se f_1(x) = o(g(x)) e f_2(x) = o(g(x)), allora

        \[ 			f_1(x) + f_2(x) = o(g(x)). 			\]

  •  

  • Il prodotto soddisfa le seguenti regole:

        \[ 			o(f(x)) \cdot o(g(x)) = o( f(x) \cdot g(x) ), \qquad 			f(x) \cdot o(g(x)) = o( f(x) \cdot g(x) ). 			\]

  •  

  • Se f(x) = o(g(x)), allora

        \[ 			o(f(x)) = o(o(g(x)) = o(g(x)). 			\]

  •  

  • Se per x \to x_0, le funzioni f_1(x) ed f_2(x) sono asintoticamente equivalentia, gli o-piccoli coincidono:

        \[ 			f_1(x) \sim f_2(x) \implies o(f_1(x)) = o(f_2(x)). 			\]

   


  1. Due funzioni F e G sono asintoticamente equivalenti se il limite \lim_{x \to + \infty} \dfrac{F(x)}{G(x)} esiste ed è uguale ad uno. Di solito, si usa il simbolo F \sim G per indicare che due funzioni soddisfano questa proprietà.

    \[\quad\]

Nel caso in cui l’argomento degli o-piccolo è una potenza di x (con esponente positivo), ad esempio nello sviluppo di Taylor, le proprietà sopra continuano a valere; tuttavia, si può dire qualcosa di più preciso:

    \[\quad\]

Algebra degli o-piccolo. Supponiamo x \to x_0=0 e prendiamo n,m>0:

  • c \cdot o(x^n) = o(c x^n) = o(x^n)
  •  

  • o(x^n) \pm o(x^m) = o(x^p), dove p := \min \{n,m\}
  •  

  • o(x^n) \cdot o(x^m) = o(x^{n+m})
  •  

  • x^m \cdot o(x^n) = o(x^{n+m})
  •  

  • x^m = o(x^n) se n<m

    \[\quad\]

Risulterà, negli esercizi, di fondamentale importanza il seguente caso particolare sull’uso di o-piccolo:

    \[\quad\]

Data una funzione f:A\subseteq \mathbb{R}\rightarrow\mathbb{R} infinitesima per x \to x_0\in \mathbb{R}\cup\{\pm \infty\}, allora, sfruttando la definizione di o-piccolo, è possibile scrivere

    \[ 					\lim_{x \to x_0}\dfrac{f(x)}{1} =0  \iff  f(x)=o(1)\quad \text{per}\,\, x \rightarrow x_0.  				\]

    \[\quad\]


 
 

Il teorema di Taylor

Introduzione.

Lo sviluppo in serie di Taylor è uno strumento fondamentale dell’analisi matematica che permette di approssimare funzioni attraverso somme di polinomi. Partendo dalle derivate di una funzione in un punto, questa tecnica consente di ottenere approssimazioni sempre più precise, trovando numerose applicazioni pratiche.

Resto nella forma di Peano.

Iniziamo la nostra discussione con l’analisi di un esempio speciale: lo sviluppo di Taylor di un polinomio che, come vedremo, ha sempre resto nullo. Consideriamo un generico polinomio di grado n

    \[ p_n(x) = a_0+a_1x+ \dots + a_n x^n,\qquad a_n \neq 0, \]

con n un intero positivo e \{a_k\}_{k=0}^n\subset\mathbb{R}. Dato x_0 \in \mathbb{R}, vogliamo riscrivere p_n(x) in un intorno di x_0, ovvero vogliamo determinare i coefficienti b_0, \dots, b_n tali che

    \[ p_n(x)=b_0+b_1(x-x_0)+ \dots +b_n(x-x_0)^n=\sum_{k=0}^{n}b_k(x-x_0)^k. \]

Valutando p_n e tutte le sue derivate (fino all’ordine n, poi si annullano) in x_0, otteniamo

    \[ \begin{cases} p_n(x_0)=b_0 \\ p'_n(x_0)=b_1, \\ \cdots \\ p^{(n)}(x_0)=n!b_n, \end{cases} \]

da cui possiamo ricavare facilmente i b_k e scrivere p_n come segue:

(4)   \begin{equation*} p_n(x)= \sum_{k=0}^n  \frac{p^{(k)}(x_0)}{k!} (x-x_0)^k. \end{equation*}

Chiaramente, per una funzione non polinomiale non si può avere una riscrittura esatta come la (4), ma bisogna tener conto di un termine di resto come esplicitato dal seguente teorema.

Teorema 1 (sviluppo di Taylor con il resto nella forma di Peano). Sia f: (a,b) \to \mathbb{R} una funzione derivabile n-1 volte in (a,b). Dato x_0 \in (a,b), supponiamo che esista f^{(n)}(x_0). Allora, per ogni x \in (a,b) si ha:

(5)   \begin{equation*} 	f(x) = p_n(x) + R_n(x;x_0), 	\end{equation*}

dove

(6)   \begin{equation*} 	p_n(x) = \sum_{k=0}^n \frac{f^{(k)}(x_0)}{k!} (x-x_0)^k 	\end{equation*}

è un polinomio di grado al più n, che prende il nome di polinomio di Taylor di f(x) centrato in x_0, mentre

(7)   \begin{equation*} 	R_n(x;x_o) = o((x-x_0)^n) \qquad \text{per $x \to x_0$} 	\end{equation*}

viene chiamato resto nella forma di Peano. Il polinomio di Taylor è l’unico polinomio con \mathrm{deg}(p_n) \le n che verifica le seguenti uguaglianze:

(8)   \begin{equation*} 	p^{(j)}_n(x_0)=f^{(j)}(x_0)\qquad \text{per ogni } j \in \{0,\ldots,n\}. 	\end{equation*}

In particolare, tutte le derivate della funzione f fino all’ordine n, valutate in x_0, coincidono con le derivate dello stesso ordine di p_n, sempre calcolate in x_0.

    \[\quad\]

Osservazione 1. Nel caso particolare in cui x_0= 0, l’espansione di Taylor per x \to 0 è anche nota in letteratura con il nome di sviluppo di McLaurin della funzione f(x).

Prima di passare alla dimostrazione del teorema 1, abbiamo bisogno di un risultato tecnico che stabilisce un criterio per dire se un polinomio è identicamente nullo.

Lemma 1. Sia P(x) un polinomio tale che \mathrm{deg}(P) \le n per qualche n \in \mathbb N. Se esiste x_0\in \mathbb{R} tale che

    \[ \lim_{x\to x_0}\dfrac{P(x)}{\left(x-x_0\right)^n}=0, \]

allora P(x) è il polinomio identicamente nullo.

    \[\quad\]

Dimostrazione. Supponiamo per assurdo che P non sia identicamente nullo, ovvero supponiamo che esista \tilde x \in \mathbb R per cui P(\tilde x) \neq 0. Dalle ipotesi del teorema sappiamo che

    \[ P(x_0) = 0, \]

ovvero x_0 è una radice di P con molteplicità k, compresa tra 1 ed n. Dunque, esiste un polinomio Q(x) tale che

    \[ P(x) = Q(x) (x-x_0)^k \quad \text{e} \quad Q(x_0) \neq 0. \]

Allora, si ha

(9)   \begin{equation*} \lim_{x\to x_0}\dfrac{P(x)}{\left(x-x_0\right)^n}=\lim_{x\to x_0}\dfrac{Q(x) \left(x-x_0\right)^k}{\left(x-x_0\right)^n}=\lim_{x\to x_0}\dfrac{Q(x)}{\left(x-x_0\right)^{n-k}}, \end{equation*}

ed osserviamo che il termine a destra della (9) diverge dato che n>k e Q(x_0) \neq 0. Tuttavia, questo è in contraddizione con l’ipotesi secondo cui il limite a sinistra è uguale a zero, concludendo la dimostrazione.

Dimostrazione del teorema 1. Per semplificare la lettura e comprensione, dividiamo la dimostrazione in tre passi: identificazione del resto di Peano, unicità del polinomio di Taylor e conclusione.

    \[\quad\]

  1. Il resto dello sviluppo fino all’ordine n si può definire come fatto nella sezione precedente:

        \[ 	R_n(x;x_0)=f(x)-\sum_{k=0}^{n}\dfrac{f^k(x_0)}{k!}\left(x-x_0\right)^k. 	\]

    Mostriamo che R_n(x;x_0) può essere espresso nella forma di Peano, ovvero soddisfa l’identità

        \[ \lim_{x \to x_0} \frac{R_n(x;x_0)}{(x-x_0)^n}=0. \]

    Sostituendo l’espressione per R_n nel limite, arriviamo ad una forma indeterminata

        \[ \lim_{x \to x_0} \frac{f(x) - \displaystyle\sum_{k=0}^n \frac{f^{(k)}(x_0)}{k!} (x-x_0)^k}{(x-x_0)^n}=\frac{[0]}{[0]}, \]

    perciò possiamo applicare il teorema di de l’Hôpital, ottenendo

        \[ \lim_{x \to x_0} \frac{R_n(x;x_0)}{(x-x_0)^n}=\lim_{x \to x_0} \frac{f'(x) - \displaystyle\sum_{k=1}^n \frac{f^{(k)}(x_0)}{(k-1)!} (x-x_0)^{k-1}}{n(x-x_0)^{n-1}}. \]

    Questa è ancora una forma indeterminata del tipo [0]/[0], pertanto si può di nuovo utilizzare il teorema di de l’Hôpital. Iterando il procedimento altre n-2 volte, si arriva a

        \[ \begin{aligned} \lim_{x \to x_0} \frac{R_n(x;x_0)}{(x-x_0)^n} & = \lim_{x \to x_0} \frac{f^{(n-1)}(x) - f^{(n-1)}(x_0) - f^{(n)}(x_0)(x-x_0)}{n!(x-x_0)} \\  & = \frac{1}{n!}\Biggl{[}\lim_{x \to x_0} \frac{f^{(n-1)}(x) - f^{(n-1)}(x_0) }{(x-x_0)}-f^{(n)}(x_0)\Biggr{]}=0, \end{aligned} \]

    dove nell’ultimo passaggio abbiamo usato l’ipotesi d’esistenza della derivata n-esima di f nel punto x_0.

  2.  

  3. Dimostriamo ora che il polinomio di Taylor è unico. Supponiamo per assurdo che esistano p_n(x)\neq q_n(x) polinomi distinti di grado minore di o uguale a n tali che

        \[ f(x)= p_n(x)+o\left(\left(x-x_0\right)^n\right) = q_n(x)+o\left(\left(x-x_0\right)^n\right),\qquad \text{per }x\to x_0. \]

    Applicando la regola su somme/sottrazioni tra o-piccoli, otteniamo

        \[ 0 = f(x) - f(x) = p_n(x)-q_n(x) + o\left(\left(x-x_0\right)^n\right) \]

    da cui si ricava immediatamente

        \[ p_n(x) - q_n(x) = o\left(\left(x-x_0\right)^n\right), \qquad \text{per } x \to x_0. \]

    Dalla definizione di o-piccolo, questo significa che il seguente limite è nullo:

        \[ \lim_{x\to x_0}\dfrac{p_n(x)-q_n(x)}{\left(x-x_0\right)^n}=0, \]

    per cui dal lemma 1 segue che il polinomio P(x):= p_n(x)-q_n(x) è identicamente nullo, ovvero p_n e q_n coincidono; questo è in contraddizione con l’ipotesi da cui siamo partiti.

  4.  

  5. Non ci resta che dimostrare la (8). Dal primo e dal secondo passo, segue che

        \[ 	f(x)=p_n(x)+o\left((x-x_0)^n\right)\qquad \text{per }x\to x_0, 	\]

    dove p_n(x) è un polinomio di grado al più n, ovvero esistono a_0,\ldots,a_n \in \mathbb R tali che

        \[p_n(x)=a_0+a_1(x-x_0)+a_2(x-x_0)^n+\dots+a_n(x-x_0)^n.\]

    Come fatto in precedenza, derivando j volte p_n(x) e valutando in x_0 si ottiene la relazione cercata

    (10)   \begin{equation*} 	f^k(x_0)=k! \; a_k, 	\end{equation*}

    e questo conclude la dimostrazione della formula di Taylor con resto di Peano.

Osservazione 2. Nell’enunciato del teorema 1 si dice che il polinomio di Taylor ha grado minore o uguale a n perché alcuni dei coefficienti (o anche tutti) possono essere nulli.

Osservazione 3. Si noti come le ipotesi scelte nell’enunciato del teorema siano minimali: viene infatti richiesto che la derivata n-esima esista1 non in un intorno, ma nel solo punto x_0. Questo significa che, nell’ultimo passo iterativo nel calcolo del limite, non è possibile applicare il teorema di de l’Hôpital ulteriormente.

Osservazione 4. Il resto è dato da

    \[ R_n(x;x_0)	=f(x) - p_n(x) = o\left(\left(x-x_0\right)^n\right), 	\]

e questo è equivalente a dire (ponendo x=x_0+h)

    \[ 	\lim_{h \to 0}\dfrac{f(x_0+h)-p_n(x_0+h)}{h^n}=0. 	\]

Perciò, il resto R_n esprime l’errore che si compie sostituendo all’espressione della funzione f il polinomio p_n ed il teorema 1 afferma che quando questo errore va a zero più velocemente di (x-x_0)^n per x\to x_0.

   


  1. L’ipotesi che la funzione sia n-1 volte derivabile in (a,b) non è necessaria; in realtà, è sufficiente richiedere che sia derivabile n-1 volte in un intorno di x_0 e n volte in x_0.

Resto nella forma di Lagrange.

Nel teorema 1, l’espressione del resto di Peano fornisce un’informazione qualitativa: esso è un infinitesimo di ordine superiore a (x-x_0)^n. In questa sezione, vogliamo trovare una versione quantitativa del resto della formula di Taylor, nota come forma di Lagrange.

Osservazione 5. L’idea è di generalizzare il teorema di Lagrange, secondo cui assegnata una funzione derivabile in un intervallo (a,b) e presi due punti x_1,x_2 \in (a,b) con x_1 < x_2, si ha:

    \[ 	f(x_2)-f(x_1)=f'(\xi)(x_2-x_1), \qquad \text{per qualche } \xi \in (x_1,x_2). 	\]

Teorema 2 (formula di Taylor con il resto in forma di Lagrange). Sia f: (a,b) \to \mathbb{R} una funzione derivabile n volte in (a,b). Dato x_0 \in (a,b), supponiamo che esista f^{(n+1)}(x) per ogni x \in (a,b)\setminus \{x_0\}. Allora per ogni x \in (a,b) esiste \xi \in (x_0, x) se x>x_0 (risp. \xi \in (x, x_0) se x< x_0) tale che

(11)   \begin{equation*} R_n(x; x_0)=\frac{f^{(n+1)}(\xi)}{(n+1)!} (x-x_0)^{n+1}. \end{equation*}

In questo caso, il termine R_n(x; x_0) viene detto resto nella forma di Lagrange.

    \[\quad\]

Dimostrazione. Senza perdere di generalità, possiamo limitarci al caso x>x_0. Definiamo

    \[ F(x) := R_n(x; x_0) = f(x)- \sum_{k=0}^n \frac{f^{(k)}(x_0)(x-x_0)^k}{k!} \]

e G(x) :=(x-x_0)^{n+1}. Portando il termine relativo a k=0 fuori dalla sommatoria, possiamo scrivere:

    \[F(x)=f(x)-f(x_0)-\sum_{k=1}^n \frac{f^{(k)}(x_0)(x-x_0)^k}{k!}.\]

Inoltre, le due funzioni appena introdotte soddisfano l’uguaglianza F(x_0)=G(x_0)=0; dunque, per il teorema di Cauchy, esiste \xi_1 \in (x_0, x) tale che

    \[ \frac{F(x)}{G(x)}=\frac{F(x)-F(x_0)}{G(x)-G(x_0)}=\frac{F'(\xi_1)}{G'(\xi_1)}. \]

Possiamo ora calcolare esplicitamente le derivate prime delle funzioni:

    \[ \begin{aligned} F^\prime(x) & = f^\prime(x)-\sum_{k=1}^{n}\dfrac{kf^{(k)}\left(x_0\right)\left(x-x_0\right)^{k-1}}{k!} \\ & =f^\prime(x)-\sum_{k=1}^{n}\dfrac{f^{(k)}\left(x_0\right)\left(x-x_0\right)^{k-1}}{\left(k-1\right)!} \\ & =f^\prime(x)-\sum_{k=0}^{n-1}\dfrac{f^{(k+1)}\left(x_0\right)\left(x-x_0\right)^{k}}{k!} \end{aligned}\]

e G'(x) = (n+1)(x-x_0)^n. Chiaramente, anche le derivate si annullano entrambe in x_0, perciò applicando di nuovo il teorema di Cauchy, troviamo \xi_2 \in (x_0, \xi_1) tale per cui

    \[ \frac{F'(\xi_1)}{G'(\xi_1)}=\frac{F'(\xi_1)-F'(x_0)}{G'(\xi_1)-G'(x_0)}=\frac{F''(\xi_2)}{G''(\xi_2)}. \]

In questo modo, possiamo iterare questo procedimento fino ad ottenere una sequenza di n punti \{ \xi_i\}_{i=1,\dots,n} ordinati (x_0<\xi_{n}\le \xi_{n-1} \le \dots \le \xi_1<x) ed un punto addizionale \xi=\xi_{n+1} \in (x_0, \xi_n) tale che:

    \[ \frac{F(x)}{G(x)}=\frac{F^{\left(n+1\right)}(\xi)}{G^{\left(n+1\right)}(\xi)}=\frac{f^{\left(n+1\right)}(\xi)}{(n+1)!} \implies F(x) = \frac{f^{\left(n+1\right)}(\xi)}{(n+1)!} G(x). \]

Di conseguenza, ricordando che F è stata definita come il resto in (x;x_0), otteniamo:

    \[ R_n(x; x_0)=F(x)=\frac{f^{(n+1)}(\xi)}{(n+1)!}G(x)=\frac{f^{(n+1)}(\xi)}{(n+1)!} (x-x_0)^{n+1}, \]

concludendo così la dimostrazione.


Resto integrale e nella forma di Cauchy.

In questa sezione presentiamo una formulazione alternativa del resto attraverso un integrale definito. Questa trattazione richiede familiarità con l’integrazione di Riemann per funzioni di una variabile reale. I lettori che non hanno ancora affrontato questi argomenti, possono procedere alla sezione successiva senza compromettere la comprensione generale. Prima di procedere, richiamiamo la seguente definizione fondamentale:

Definizione 2. Sia \Omega un sottoinsieme aperto di \mathbb{R} e k\in\mathbb{N}. Una funzione

    \[ f : \Omega \to \mathbb R \]

si dice di classe C^k(\Omega) se in ogni punto di \Omega esistono tutte le derivate di f fino al k-esimo ordine, e tali derivate sono continue.

Inoltre, una funzione f:\Omega \to \mathbb R si dice di classe C^\infty(\Omega) (o liscia) se in ogni punto di \Omega esistono tutte le derivate di qualsiasi ordine, e tali derivate sono funzioni continue.

    \[\quad\]

Osservazione 6. Una funzione f è liscia se e solo se f \in C^k(\Omega) per ogni k \in \mathbb N. In altre parole, si ha

    \[ 	C^\infty(\Omega) = \bigcap_{k=0}^\infty C^k(\Omega). 	\]

Teorema 3 (resto in forma integrale). Sia f: (a,b) \to \mathbb{R} una funzione di classe C^{n+1}. Dato x_0 \in (a,b), per ogni x \in (a,b) si ha:

(12)   \begin{equation*} R_n(x; x_0)=\int_{x_0}^x \frac{(x-\lambda)^{n}}{n!}f^{(n+1)}(\lambda)   \, d\lambda. \end{equation*}

In questo caso, il termine R_n(x; x_0) viene detto resto in forma integrale.

    \[\quad\]

Fissiamo x \in (a,b) e consideriamo il resto R_n(x;x_0) come funzione del solo punto x_0 \in (a,b). In altre parole, introduciamo la funzione ausiliaria di una variabile reale:

(13)   \begin{equation*}  	H(x_0) := R_n(x;x_0) = f(x)-\sum_{k=0}^{n}\dfrac{f^{(k)}(x_0)\left(x-x_0\right)^k}{k!}=f(x)-f(x_0)-\sum_{k=1}^{n}\dfrac{f^{(k)}(x_0)\left(x-x_0\right)^k}{k!}. \end{equation*}

Dall’ipotesi di regolarità fatta su f e dall’uguaglianza (13) segue che H(x_0) può essere derivata rispetto a x_0; inoltre, tutti i termini di ordine inferiore ad n si elidono a due a due:

    \[ \begin{aligned} \dfrac{d H(x_0) }{d x_0}&=\underbrace{\dfrac{df(x)}{dx_0}}_{=0}-f^{\prime}(x_0)-\sum_{k=1}^{k}\dfrac{d\left(\dfrac{f^{(k)}(x_0)\left(x-x_0\right)^k}{k!}\right)}{d x_0}=\\ &=-f^{\prime}(x_0)-\sum_{k=1}^{n}\left(\dfrac{f^{(k+1)}\left(x_0\right)\left(x-x_0\right)^{k}}{k!}-\dfrac{f^{(k)}(x_0)\left(x-x_0\right)^{k-1}}{\left(k-1\right)!}\right)=\\ &=-f^{\prime}(x_0)+\left(\underbrace{-f^{\prime\prime}\left(x_0\right)\left(x-x_0\right)+f^\prime\left(x_0\right)}_{k=1}\right)+\left(\underbrace{-\dfrac{f^{\prime\prime \prime }\left(x_0\right)\left(x-x_0\right)^2}{2!}+f^{\prime\prime}\left(x_0\right)\left(x-x_0\right)}_{k=2}\right)+\\ &+\underbrace{-\dfrac{f^{(4)}\left(x_0\right)\left(x-x_0\right)^3}{3!}+\dfrac{f^{\prime\prime\prime}\left(x_0\right)\left(x-x_0\right)^2}{2!}}_{k=3}\dots +\underbrace{-\dfrac{f^{(n+1)}\left(x_0\right)\left(x-x_0\right)^n}{n!}+\dfrac{f^{\left(n\right)}\left(x_0\right)\left(x-x_0\right)^{n-1}}{\left(n-1\right)!}}_{k=n}=\\ &=-\dfrac{f^{\left(n+1\right)}\left(x_0\right)\left(x-x_0\right)^n}{n!}. \end{aligned} \]

Ricordiamo inoltre che, data una funzione continua h in (a,b), il teorema fondamentale del calcolo integrale ci dice che per ogni x \in (a,b) si ha

    \[ h(x)-h(a)=\int_{a}^x h'(\lambda) \, d\lambda. \]

Dalle ipotesi di regolarità su f, la funzione H è continua; pertanto, soddisfa la seguente uguaglianza:

    \[ R_n(x;x) - R_n(x;x_0)= \int_{x_0}^x \dfrac{dH(\lambda) }{d \lambda} \, d\lambda=-\int_{x_0}^x  \frac{(x-\lambda)^n}{n!}f^{(n+1)}(\lambda) \,d\lambda. \]

Dato che il resto è nullo quando valutato in entrambi gli argomenti nello stesso punto, ovvero R_n(x;x)=0, sostituendo nella uguaglianza sopra otteniamo:

    \[ \underbrace{R_n(x;x)}_{=0} - R_n(x;x_0)=-\int_{x_0}^x  \frac{(x-\lambda)^n}{n!}f^{(n+1)}(\lambda) \,d\lambda \implies  R_n(x;x_0)=\int_{x_0}^x  \frac{(x-\lambda)^n}{n!}f^{(n+1)}(\lambda) \,d\lambda, \]

concludendo così la dimostrazione.

Il resto in forma integrale (13) permette di ottenere una nuova espressione puntuale del resto, che ha una forma molto simile a quella di Lagrange descritta nella sezione precedente.

Corollario 1 (resto in forma di Cauchy). Sia f: (a,b) \to \mathbb{R} una funzione di classe C^{n+1}. Dato x_0\in(a,b), per ogni x \in (a,b) esiste \xi \in (x_0, x) se x>x_0 (risp. \xi \in (x, x_0) se x_0>x) tale che

(14)   \begin{equation*} R_n(x; x_0)=\frac{f^{(n+1)}(\xi)(x-\xi)^n}{n!}(x-x_0). \end{equation*}

In questo caso, il termine R_n(x; x_0) viene detto resto in forma di Cauchy.

    \[\quad\]

Osservazione 7. Un ingrediente fondamentale per la dimostrazione è il teorema della media integrale, secondo cui, data una funzione continua h: (a,b) \to \mathbb{R}, esiste \xi \in (a, b) tale che

(15)   \begin{equation*} 		\frac{1}{b-a} \int_{a}^b h(t) \, dt=h(\xi).  		\end{equation*}

Dimostrazione del corollario 1. Senza perdere di generalità, possiamo metterci nel caso in cui x>x_0. Consideriamo la funzione

    \[ h(t) := \frac{f^{(n+1)}(t)(x-t)^n}{n!} \]

che si trova sotto il segno di integrale in (12), ed applichiamo (15) nell’intervallo (x_0,x). Possiamo dunque trovare un punto \xi \in (x_0,x) tale per cui

    \[ R_n\left(x,x_0\right)=\int_{x_0}^{x} h(t) \, dt = h(\xi) = \dfrac{f^{\left(n+1\right)}\left(\xi\right)\left(x-\xi\right)^n}{n!}\left(x-x_0\right), \]

da cui segue la tesi.

Osservazione 8. Si può ricavare anche l’espressione del resto in forma di Lagrange applicando il teorema della media generalizzato alla forma integrale del resto in (12).

Si ricorda infatti che, data una funzione integrabile \phi di segno costante ed una funzione \psi continua nell’intervallo (a,b), il teorema della media generalizzato afferma che esiste \xi \in (a, b) tale che

    \[  \int_{a}^b \phi(t)\psi(t) \, dt=\psi(\xi)  \int_{a}^b \phi(t) \, dt. \]

Il resto in forma di Lagrange si trova a questo punto applicando il teorema con la scelta \psi(x)=f^{(n+1)}(x) e \phi(x)=(x-x_0)^n/n!. Infatti, un semplica calcolo ci mostra che

    \[ \begin{aligned} \int_{x_0}^{x}\dfrac{\left(t-x_0\right)^nf^{\left(n+1\right)}\left(t\right)}{n!}\,dt & = f^{\left(n+1\right)}\left(\xi\right)\int_{x_0}^{x}\dfrac{\left(t-x_0\right)^n}{n!}\,dt \\ & =f^{\left(n+1\right)}\left(\xi\right)\left(\dfrac{\left(t-x_0\right)^{n+1}}{\left(n+1\right)!}\right)\bigg\vert^{x}_{x_0} \\& =\dfrac{f^{(n+1)}\left(\xi\right)\left(x-x_0\right)^{n+1}}{\left(n+1\right)!}. \end{aligned} \]


Serie di Taylor e funzioni analitiche.

Supponiamo di avere una funzione liscia, ovvero f \in C^{\infty}\left(\left(a,b\right)\right). Dato x_0 \in (a,b), il teorema 1 ci dice che

    \[ f(x)=\sum_{k=0}^n \frac{f^{(k)}(x_0)}{k!}(x-x_0)^k + R_n(x; x_0) \]

è verificata per ogni n \in \mathbb N. Viene dunque spontaneo chiedersi per quali x \in (a,b) queste somme parziali convergano al valore f(x) quando n \to + \infty. In altre parole, ci chiediamo per quali x \in (a,b) si ha

    \[ \lim_{n \to + \infty} R_n(x;x_0) = 0. \]

Questa domanda spontanea porta all’introduzione di una nuova classe di funzioni, dette funzioni analitiche, che rappresentano un oggetto fondamentale nell’analisi matematica. Ricordiamo che vale l’equivalenza:

    \[ \begin{gathered} \sum_{n=0}^{+\infty}\dfrac{f^{\left(n\right)}\left(x_0\right)}{n!}\left(x-x_0\right)^n = f(x) \\ \Leftrightarrow \\ \lim_{n\to +\infty} R_n(x; x_0)=\lim_{n\to +\infty}\left(f(x)-\sum_{k=0}^n \frac{f^{(k)}(x_0)}{k!}(x-x_0)^k\right)=0. \end{gathered} \]

Definizione 3. Siano I\subseteq \mathbb{R} un intervallo aperto, f \in C^{\infty}(I) e x_0 \in I. Diciamo che la funzione f è sviluppabile in serie di Taylor in x_0 se esiste un intervallo aperto J \subseteq I, con x_0 \in J, tale che

(16)   \begin{equation*} f(x)= \sum_{k=0}^\infty\frac{ f^{(k)}(x_0)(x-x_0)^k}{k!}, \qquad \text{per ogni } x \in J. \end{equation*}

L’intervallo massimale J per cui vale (16) viene detto intervallo di convergenza della serie. Inoltre una funzione f\in C^{\infty}\left(I\right) si dice analitica nell’intervallo I se \forall x_0 \in I\,\,\exists\delta >0:\forall x \in \left(x_0-\delta ,x_0+\delta \right) vale (16).

    \[\quad\]

Definizione 4. Una funzione f\in C^{\infty}\left(I\right) si dice analitica nell’intervallo I se f è sviluppabile in serie di Taylor nel senso della definizione 3 in ogni x_0 \in I.

Esempio 1 (una funzione C^{\infty} ma non analitica). E’ importante osservare che f \in C^\infty(I) è una condizione necessaria, ma non sufficiente, per l’analiticità. Consideriamo, ad esempio, la funzione definita a tratti:

    \[ f(x)= \begin{cases} \mathrm{e}^{-\frac{1}{x}} & \text{se $x >0$}\\ 0 & \text{se $x \le 0$}. \end{cases} \]

La funzione è continua in x=0, infatti \displaystyle \lim_{x \to 0^+} \mathrm{e}^{-\frac{1}{x}}=0=f(0). Inoltre, la derivata prima è data da

    \[ f'(x)= \begin{cases} x^{-2} \mathrm{e}^{-\frac{1}{x}} & \text{se $x >0$}\\ 0 & \text{se $x \le 0$}, \end{cases} \]

dove abbiamo usato che

    \[ f'(0^+)=\lim_{h \to 0^+} \frac{\mathrm{e}^{-\frac{1}{h}}}{h^2}=0, \]

ovvero f è derivabile in x=0. Iterando il procedimento, si può dimostrare che per x>0 e k \in \mathbb N si ha

    \[ f^{(k)}(x)=\mathrm{e}^{-\frac{1}{x}}p_{2k}\biggl{(}\frac{1}{x}\biggr{)}, \]

dove p_{2k}(x) è un generico polinomio di grado 2k con termine noto nullo. Dato che

    \[ \lim_{x \to 0^+} x^{-N} \mathrm{e}^{-\frac{1}{x}} =0 \qquad \text{per ogni } N \in \mathbb N, \]

deduciamo che la funzione è di classe C^\infty(\mathbb R) e f^{(k)}(0)=0 per ogni k \ge 0. Di conseguenza, lo sviluppo di Taylor in x_0=0 ci da

    \[ \sum_{k=0}^\infty \frac{\overbrace{f^{(k)}(0)}^{=0}}{k!} x^k = 0, \]

quindi f(x) non coincide con il suo sviluppo di Taylor in un qualsiasi intorno destro dell’origine dato che

    \[ 0< \mathrm{e}^{-\frac{1}{x}} \neq \sum_{k=0}^\infty \frac{f^{(k)}(0)}{k!} x^k=0. \]


 
 

Sviluppi di funzioni elementari

Introduzione.

In questa sezione, presentiamo alcuni sviluppi di McLaurin noti di funzioni elementari (i.e., funzioni polinomiali, razionali, trigonometriche, esponenziali e iperboliche, nonché le somme, i prodotti, le composizioni e le inverse di queste). Ci concentriamo, inoltre, sullo stabilire gli intervalli di convergenza delle serie di Taylor.

La funzione esponenziale.

Consideriamo la funzione f(x)=\mathrm{e}^x e determiniamone lo sviluppo di Taylor intorno al punto x_0=0. Poiché la derivata della funzione esponenziale è la funzione esponenziale stessa, si ha che

    \[ f^{(k)}(x)=\mathrm{e}^x  \quad \text{e} \quad  f^{(k)}(0)=1 \]

per ogni k \in \mathbb N. Pertanto, per il teorema 1, per ogni n \in \mathbb N e si ha

    \[ \mathrm{e}^x=\sum_{k=0}^n \frac{x^k}{k !}+\frac{\mathrm{e}^{\xi}}{(n+1)!}x^{n+1}, \]

dove \xi \in (0,x) oppure \xi \in (x,0) in base al segno della x. Osserviamo che

    \[ \left| \frac{\mathrm{e}^{\xi}}{(n+1)!}x^{n+1}\right| \le\frac{ \mathrm{e}^{|x|}|x|^{n+1}}{(n+1)!} \qquad \text{per ogni } n \in \mathbb N \text{ ed ogni } x \in \mathbb R.\]

Essendo x^n=o(n!) per n \to \infty si ha che

    \[  \lim_{n \to \infty} \frac{ \mathrm{e}^{|x|}|x|^{n+1}}{(n+1)!}=0  \]

da cui, per il teorema del confronto, si deduce che:

    \[ \lim_{n \to \infty}\left \vert \frac{\mathrm{e}^{\xi}}{(n+1)!}x^{n+1}\right \vert=0 \qquad \text{per ogni } x \in \mathbb{R}. \]

Dunque, la funzione f è sviluppabile in serie di Taylor in x_0= 0 (secondo la definizione 3) ed ha intorno di convergenza che coincide con tutto \mathbb R. Analogamente, si dimostra che

    \[ e^x = \sum_{k = 0}^\infty \frac{e^{x_0}}{k!} (x-x_0)^k \]

per ogni x_0 \in \mathbb R, da cui segue che f(x)=e^x è una funzione analitica (definizione 4) su tutto \mathbb R.


La funzione logaritmica.

Consideriamo la funzione f(x)=\ln(1+x) e cerchiamo lo sviluppo in serie di Taylor in x_0 = 0. Ricordando che

    \[ f'(x) = \frac{1}{1+x}, \]

è facile verificare per induzione che per ogni n \ge 1 si ha:

    \[ f^{(n)}(x)=(-1)^{n+1}\frac{(n-1)!}{(1+x)^{n}}. \]

Calcolando la derivata n-esima in x_0=0 si trova

    \[ f^{(n)}(0) = (-1)^{n+1} (n-1)!. \]

In questo caso, il resto in forma di Lagrange è dato da

    \[ R_{n}\left(x;0\right)=\dfrac{x^{n+1}f^{\left(n+1\right)}\left(\xi\right)}{\left(n+1\right)!}=\dfrac{\left(-1\right)^n x^{n+1}}{\left(1+\xi\right)^{n+1}\left(n+1\right)}, \]

ma, come il lettore può verificare in dettaglio per esercizio, non è sufficientemente preciso e consente di trovare un intervallo di convergenza che non risulta essere quello massimale. Infatti, scrivendo

    \[ \begin{aligned} \ln(1+x) & = \sum_{k=1}^{n} \frac{(-1)^{k+1}(k-1)!}{k!}x^k +(-1)^{n} \frac{x^{n+1}}{(1+\xi)^{n+1}(n+1)} \\ & =\sum_{k=0}^{n-1} (-1)^{k}\frac{x^{k+1}}{k+1}+(-1)^{n} \frac{x^{n+1}}{(1+\xi)^{n+1}(n+1)}, \end{aligned} \]

possiamo dimostrare che il resto tende a zero quando x \in [-1/2,1], ma non riusciamo a dire nulla in (-1,-1/2) e questo non è soddisfacente. L’idea è quella di dividere lo studio in due casi:

    \[\quad\]

  • Se x>0, allora 0<\xi<x e, poiché in tal caso (1+\xi)^{-1}<1, si ha

        \[ 0 \le \frac{x^{n+1}}{(1+\xi)^{n+1}(n+1)} \le \frac{x^{n+1}}{n+1}\qquad \text{per ogni } n \in \mathbb{N}. \]

    Il termine a destra tende a zero per n \to +\infty se e solo se x\in [0,1] (avendo ristretto l’intervallo ai soli x positivi!). Dunque, per il teorema del confronto, il resto tende a zero per n\to +\infty quando x\in [0,1].

  •  

  • Se x < 0, allora x<\xi<0 e, poiché in tal caso 1+\xi>1+x, si ha

        \[ \left \vert \frac{x^{n+1}}{(1+\xi)^{n+1}(n+1)} \right \vert \le \frac{|x|^{n+1}}{(1+x)^{n+1}(n+1)}=\left(\frac{|x|}{1+x} \right)^{n+1} \frac{1}{n+1}, \]

    che tende a zero per n \to \infty se e solo se

        \[ -1 \le \dfrac{\left \vert x\right \vert}{1+x}\leq 1. \]

    Tuttavia, dato che stiamo considerando il caso x < 0, il modulo si può sostituire con -x e la condizione diventa:

        \[ -1 \le \frac{-x}{x+1}\le 1 \iff x\in [-1/2,+\infty), \]

    e questo ci fa concludere che il resto tende a zero per n\to +\infty quando x \in [-1/2,0).

Riassumendo abbiamo dimostrato che il resto tende a zero per n\to +\infty se x \in [-1/2,1], ma questo non coincide con l’intervallo massimale che è dato da

    \[ I_{max} = (-1,1]. \]

L’idea è dunque quella di utilizzare, nel caso x<0, un resto più preciso, ad esempio quello in forma di Cauchy:

    \[ R_n(x; 0)=\frac{xf^{(n+1)}(\xi)(x-\xi)^n}{n!}=\dfrac{x\left(x-\xi\right)^n\left(-1\right)^n}{\left(1+\xi\right)^{n+1}}=\dfrac{x\left(-1\right)^n}{1+\xi}\left(\dfrac{x-\xi}{1+\xi}\right)^n. \]

Sia x\in (-1,0). Allora -1<x<\xi <0 e, ricordando che la funzione -(1+\xi)^{-1} nell’intervallo considerato è crescente e ha massimo in \xi=0, si ha

    \[ \left \vert \dfrac{x-\xi}{1+\xi}\right \vert=\dfrac{\xi-x}{1+\xi}=1-\dfrac{1+x}{1+\xi}<1-1-x=-x=\left \vert x \right \vert. \]

Di conseguenza, abbiamo

(17)   \begin{equation*} 0 \le \left| \dfrac{x\left(-1\right)^n}{1+\xi}\left(\dfrac{x-\xi}{1+\xi}\right)^n\right| \le \left| \frac{x}{1+\xi} \right| \cdot \left| \frac{x-\xi}{1+\xi} \right|^n < \dfrac{\left|x\right|^{n+1}}{1+\xi} \xrightarrow{n\to+\infty} 0. \end{equation*}

In conclusione, la funzione f è sviluppabile in serie di Taylor centrata in x_0=0 e con intervallo massimale2 di convergenza dato da I_{max} = (-1,1].

   


  1. E’ un esercizio banale quello di verificare che la serie di Taylor diverge in (-\infty,-1] \cup (1,+\infty).

Le funzioni seno e coseno.

Consideriamo la funzione trigonometrica f(x)=\sin(x). Siamo interessati allo sviluppo in x_0 = 0, perciò iniziamo con l’osservare che la derivata k-esima è data da

    \[ f^{(k)}(x)= \sin\left(\dfrac{\pi}{2}k\right)\cos x +\cos\left(\dfrac{\pi}{2}k\right)\sin x, \]

da cui segue che valutata in x_0 essa vale:

    \[ f^{\left(k\right)}\left(0\right)=\sin\left(\dfrac{\pi}{2}k\right). \]

Notiamo che, in particolare, la funzione \sin(x) è periodica di periodo 2\pi ed è uguale a zero in 0 e \pi, perciò per i valori pari di k=2m abbiamo

    \[ f^{(2m)}(0) = \sin (\pi) = 0, \]

da cui segue che la serie di Taylor ha soltanto i termini con indice dispari diversi da zero. Inoltre, per k = 2m+1 (dispari) si ha

    \[ f^{(2m+1)}(0) = \sin \left(m \pi + \frac\pi2 \right) = (-1)^m. \]

Mettendo insieme queste due osservazioni, possiamo scrivere lo sviluppo in serie della funzione seno come:

    \[ \sin(x)=\sum_{m=0}^{n-1} \frac{(-1)^{m}}{(2m+1)!}x^{2m+1}+R_{2n}(x;0). \]

Esprimendo il resto nella forma di Lagrange, abbiamo che esiste \xi in un opportuno intervallo (in base al segno della x) tale per cui si può scrivere:

    \[ R_{2n}(x;0)=(-1)^{n-1}\frac{x^{2n+1}}{(2n+1)!}\cos(\xi). \]

Dato che la funzione coseno è limitata (-1 \le \cos x \le 1), abbiamo la stima seguente per il resto:

    \[  |R_{2n}(x;0)| \le\frac{|x|^{2n+1}}{(2n+1)!}. \]

Essendo x^n=o(n!) per n \to \infty, concludiamo che \displaystyle\lim_{n \to \infty} R_{2n}(x; x_0)=0 per ogni x \in \mathbb{R}. Dunque la funzione è sviluppabile in serie di Taylor in x_0 = 0 con intervallo massimale \mathbb R (secondo la definizione 3). Inoltre, come il lettore può verificare per esercizio, si ha

    \[ \sin(x) = \sum_{n=0}^{+ \infty} \frac{\sin \left( \frac{n \pi}{2} + x_0 \right)}{n!} (x-x_0)^n \qquad \text{per ogni } x_0 \in \mathbb R, \]

e dunque la funzione f(x) = \sin(x) è analitica su tutto \mathbb R (definizione 4). Analogamente, se consideriamo la funzione trigonometrica g(x)=\cos(x), si può verificare per induzione che

    \[ g^{(k)}(x)= -\sin\left(\dfrac{\pi}{2}k\right)\sin x +\cos\left(\dfrac{\pi}{2}k\right)\cos x. \]

Valutando in x_0=0 e procedendo allo stesso modo di quanto fatto con \sin x, osserviamo che lo sviluppo di g contiene solo i termini di grado pari e si può scrivere come segue:

    \[ \cos(x)=\sum_{k=0}^{n-1} (-1)^{k}\frac{x^{2k}}{(2k)!}+R_{2n-1}(x;0). \]

Esprimendo il resto in forma di Lagrange come fatto in precedenza, si trova

    \[ R_{2n-1}(x;0)=(-1)^{n}\frac{x^{2n}}{(2n)!}\cos(\xi) \implies  |R_{2n-1}(x;0)| \le\frac{|x|^{2n}}{(2n)!}, \]

da cui si conclude che che \displaystyle\lim_{n \to \infty} R_{2n-1}(x;0)=0 per ogni x \in \mathbb R. Inoltre, si può verificare che

    \[ \cos(x) = \sum_{n=0}^{+\infty} \frac{\cos\left(\dfrac{\pi n}{2} + x_0\right)}{n!}(x - x_0)^n \qquad \text{per ogni } x_0 \in \mathbb R,  \]

e questo significa che la funzione g(x) = \cos x è analitica su tutto \mathbb R.


La serie geometrica e la funzione arcotangente.

Consideriamo la funzione f(x)= 1/(1-x) e cerchiamo lo sviluppo in serie Taylor in un intorno di x_0=0. Si può verificare per induzione che la derivata k-esima è data da:

    \[ f^{(k)}(x)=\frac{k!}{(1-x)^{k+1}} \implies   f^{(k)}(0)=k!, \]

da cui, per la formula di Taylor (5), si ottiene la scrittura

    \[ \frac{1}{1-x}= \sum_{k=0}^n x^k + R_n(x;0). \]

Osserviamo subito che la funzione f(x) non è definita in x=1, perciò ogni intorno di x_0=0 è necessariamente limitato a destra da x=1. In particolare, ci limitiamo a studiare la convergenza in (-\infty,0).

Osservazione 9. Tale sviluppo si può ottenere senza ricorrere all’espansione in serie di Taylor. Infatti, iniziamo considerando le somme parziali della serie geometrica:

    \[ S_n(x) = 1+ \dots + x^n. \]

Moltiplicando a destra e sinistra per x (si noti che x può essere uguale a zero), otteniamo

    \[ xS_n(x)=x+ \dots+ x^{n+1}, \]

e una semplice sottrazione ci da

    \[ (1-x)S_n(x)=1-x^{n+1}. \]

A questo punto, supponendo x \neq 1, possiamo dividere ed ottenere una identità per le somme parziali:

    \[ S_n (x) = \sum_{k=0}^{n}x^k = \dfrac{1-x^{n+1}}{1-x}, \]

e questo tramite una semplice manipolazione algebrica ci permette di ottenere un’espressione esplicita per il resto R_n(x;0):

(18)   \begin{equation*} \frac{1}{1-x}=S_n(x) + \Biggl{(} \frac{1}{1-x} - S_n(x) \Biggr{)}=S_n\left(x\right)+\left(\dfrac{1}{1-x}-\dfrac{1-x^{n+1}}{1-x}\right)=\sum_{k=0}^n x^k + \underbrace{\frac{x^{n+1}}{1-x}}_{=R_n(x;0)}. \end{equation*}

Dunque, il resto ottenuto in (18) tende a zero per n \to + \infty nell’intervallo I = (-1,1): la funzione f è sviluppabile in serie di Taylor centrata in x_0=0 con intervallo di convergenza dato da I.

Osservazione 10. Dato che l’intervallo (-1,1) è simmetrico rispetto l’origine, possiamo applicare la trasformazione x \mapsto -x, ottenendo un altro sviluppo molto utile:

    \[ 	\frac{1}{1+x}= \sum_{k=0}^n (-1)^k x^k + (-1)^{n+1}\frac{ x^{n+1}}{1+x}. 	\]

La funzione f(x) = 1/(1+x) è continua, e dunque integrabile, nell’intervallo (-1,+\infty). Analogamente, il termine a destra è dato dalla somma di funzioni continue sullo stesso intervallo, perciò possiamo integrare:

(19)   \begin{equation*} 	\int_0^x \frac{1}{1+s} \, ds=\log(1+x)= \sum_{k=0}^n (-1)^k \frac{x^{k+1}}{k+1} + \int_0^x (-1)^{n+1}\frac{ s^{n+1}}{1+s} \, ds, \qquad \text{per ogni } x \in (-1,+\infty). 	\end{equation*}

Facendo ragionamenti analoghi a quelli fatti in (17) si dimostra che il resto tende a zero per x \in (-1,1] e, dunque, che lo sviluppo del logaritmo centrato in x_0=0 è dato da

    \[ \log(1+x)=\sum_{k=0}^{+\infty} (-1)^k \frac{x^{k+1}}{k+1}, \qquad \text{per } x \in (-1,1]. \]

Osservazione 11. Dato che l’intervallo (-1,1) è simmetrico rispetto l’origine, possiamo applicare la trasformazione x \mapsto -x^2, ottenendo un altro sviluppo interessante:

    \[ 	\frac{1}{1+x^2}= \sum_{k=0}^n (-1)^k x^{2k} + (-1)^{n+1}\frac{x^{2(n+1)}}{1+x^2}. 	\]

Analogamente a quanto detto nell’osservazione precedente, abbiamo a destra e sinistra due funzioni continue (dunque integrabili) in tutto l’insieme dei numeri reali \mathbb R. Ne segue che:

(20)   \begin{equation*} \arctan(x) = \int_0^x  \frac{1}{1+s^2} \, ds =\sum_{k=0}^n (-1)^k \frac{x^{2k+1}}{2k+1} + R_n(x;0), \end{equation*}

dove

    \[ \begin{aligned}  |R_n(x;0)| & \le \int_0^{x} \left| \frac{s^{2n+2}}{1+s^2} \right| \, ds  \\ & \le \int_{0}^{x}\left| s^{2n+2} \right| \,  ds \\ & = \int_{0}^{x}\left| s \right|^{2n+2}\, ds =\frac{|x|^{2n+3}}{2n+3}\xrightarrow{n\to+\infty} 0 \qquad \forall x \in [-1,1]. \end{aligned}\]

Di conseguenza, la funzione \arctan(x) è sviluppabile in serie di Taylor con centro x_0=0 ed intervallo di convergenza I = [-1,1].


Il binomio di Newton generalizzato.

Consideriamo la funzione che generalizza il binomio di Newton, ovvero

    \[ f(x)=(1+x)^{\alpha} \]

con \alpha che può prendere valori in tutto \mathbb R e studiamo lo sviluppo in serie di Taylor in x_0 = 0. Osserviamo che nel caso \alpha \in \mathbb N si ha

    \[ f(x) = (1+x)^n = \sum_{k=0}^n \binom{n}{k} x^k, \]

e trattandosi di un polinomio f coincide con il suo sviluppo di Taylor. Nel caso generale, ovvero \alpha \in \mathbb R, questo non è più vero; nello specifico, iniziamo con il calcolare la derivata k-esima di f:

    \[ f^{(k)}(x)=  (1+x)^{\alpha - k} \prod_{j=0}^{k-1} (\alpha - j). \]

Questa formula, valida per ogni \alpha, si può verificare facilmente per induzione su k. Dunque, si ha

    \[ f^{(k)}(0)= \prod_{j=0}^{k-1} (\alpha - j), \]

Sviluppando in serie di Taylor si trova

    \[ \begin{aligned} (1+x)^\alpha & =1+ \sum_{k=1}^{n} \Biggl{(} \prod_{j=0}^{k-1} (\alpha - j) \Biggr{)}\frac{x^{k}}{k!}+R_{n}(x;0) \\ & =1+\sum_{k=1}^{n}\dfrac{\alpha\left(\alpha-1\right)\dots \left(\alpha-\left(k-1\right)\right) }{k!}\cdot x^k+R_n\left(x,0\right), \end{aligned} \]

che possiamo riscrivere come

(21)   \begin{equation*}   (1+x)^\alpha=\sum_{k=0}^{n} \binom{\alpha}{k}x^{k}+R_{n}(x;0), \end{equation*}

dove, per comodità di notazione, abbiamo introdotto il coefficiente binomiale generalizzato:

    \[  \binom{\alpha}{k}= \frac{\alpha(\alpha-1) \cdots (\alpha-k+1)}{k!}. \]

Per dimostrare che la funzione f(x)=(1+x)^\alpha è sviluppabile in serie di Taylor centrata in x_0=0 (definizione 3) con intervallo di convergenza I=(-1,1), iniziamo con l’enunciare un risultato tecnico piuttosto utile:

Lemma 2 Sia |x|<1. Allora per ogni \alpha \in \mathbb R esistono N \in \mathbb N e q \in [0,1] tali che

    \[ \left \vert \left(1-\dfrac{\alpha}{k}\right)x\right \vert \leq q, \qquad \text{per ogni } k \ge N. \]

    \[\quad\]

Dimostrazione. Applicando la diseguaglianza triangolare al termine a sinistra otteniamo

(22)   \begin{equation*} 		\left \vert\left(1-\dfrac{\alpha}{k}\right)x \right \vert\leq \left(1+\dfrac{\left \vert \alpha\right \vert }{k}\right)\left \vert x \right \vert \qquad \forall \alpha \in \mathbb{R},\,\forall 	x \in (-1,1) \,\,\text{e}\,\,k \in \mathbb{N}\setminus\{0\}. 		\end{equation*}

Se 0<|x|<1, allora 1/|x|>1 e si vede subito che

    \[ 		\begin{aligned} 		\left(1+\dfrac{\left \vert a \right \vert}{k}\right)\left \vert x \right \vert <1 & \iff 1+\dfrac{\left \vert \alpha \right \vert}{k}<\dfrac{1}{\left \vert x \right \vert }\iff \dfrac{\left \vert \alpha \right \vert }{k}<\dfrac{1}{\left \vert x \right \vert}-1 \\ 		& \iff k>\dfrac{\left \vert \alpha \right \vert }{\dfrac{1}{\left \vert x \right \vert }-1}=\dfrac{\left \vert \alpha x \right \vert }{1-\left \vert x \right \vert } 		\end{aligned} 		\]

per ogni \alpha \in \mathbb R ed ogni x \in (-1,1).

Utilizziamo la funzione parte intera, ovvero \floor{x}:=\max\lbrace k\in\mathbb{Z}\colon k\le x\rbrace, per definire il numero intero N=\floor{ |\alpha x| / (1-|x|)}+1; si ha

    \[ 				q:= \left(1+\dfrac{\left \vert \alpha \right \vert}{N}\right) \left \vert x \right \vert <1\qquad \forall k>N \text{ e } \forall 	x \in (-1,1), 		\]

da cui segue che

    \[ 		\left \vert \left(1-\dfrac{\alpha }{k}\right)x \right \vert \leq  q \qquad \forall k>N \text{ e } \forall 	x \in (-1,1), 		\]

concludendo così la dimostrazione del lemma.

Possiamo ora stimare il resto esprimendolo nella forma di Cauchy (14). A tal proposito, iniziamo con il calcolare la derivata f^{\left(k+1\right)} in un generico punto x:

    \[  f^{\left(k+1\right)}\left(x\right)=\left(\alpha - k\right)\left(1+x\right)^{\alpha-k-1}\prod_{j=0}^{k-1}\left(\alpha -j\right)=\left(1+x\right)^{\alpha-k-1}\prod_{j=0}^{k	}\left(\alpha -j\right).  \]

Di conseguenza, esiste \xi \in \mathbb R con |\xi|<|x| tale per cui

    \[ \begin{aligned} R_{n}(x;0) & = \frac{1}{n!}\Biggl{(}\prod_{j=0}^{n} (\alpha - j) \Biggr{)}(1+\xi)^{\alpha - n-1}(x-\xi)^nx=\\ &=\frac{\alpha}{n!}\Biggl{(}\prod_{j=1}^{n} \left(\alpha - j\right) \Biggr{)}(1+\xi)^{\alpha -1}\left(1+\xi\right)^{-n}x^{n}\left(1-\dfrac{\xi}{x}\right)^nx=\\ &=\dfrac{\alpha}{n!}\left(\left(\left(-1\right)\left(1-\dfrac{\alpha}{1}\right)\right)\dots\left(\left(-n\right)\left(1-\dfrac{\alpha}{n}\right)\right)\right)\left(1+\xi\right)^{\alpha-1}\left(\dfrac{1-\dfrac{\xi}{x}}{1+\xi}\right)^nx^{n+1}=\\ &=\dfrac{\left(-1\right)^n\alpha \,n!}{n!}\left(\prod_{j=1}^{n}\left(1-\dfrac{\alpha}{j}\right)\right)\left(1+\xi\right)^{\alpha-1}\left(\dfrac{1-\dfrac{\xi}{x}}{1+\xi}\right)^nx^{n+1}\\ &=(-1)^{n}\Biggl{(}\alpha\prod_{j=1}^{n} \Bigl{(} 1-\frac{\alpha}{j} \Bigr{)} \Biggr{)}(1+\xi)^{\alpha-1}\left(\frac{1-\dfrac{\xi}{x}}{1+\xi}\right)^nx^{n+1}, \end{aligned} \]

dove abbiamo usato la seguente proprietà algebrica:

    \[ \prod_{j=1}^{n} \left(\alpha - j\right)=(-1)^n n!\prod_{j=1}^{n} \left(1-\frac{\alpha}{j}\right). \]

Mettiamoci ora nel caso 0<\xi<x<1. Si ha

    \[ \dfrac{1-\dfrac{\xi}{x}}{1+\xi}<1 \iff 1-\dfrac{\xi}{x}<1+\xi \iff x-\xi<x+x\xi \iff -1<x\iff x>-1\quad \checkmark \]

Analogamente, nel caso in cui -1<x<\xi < 0, si ha

    \[ \dfrac{1-\dfrac{\xi}{x}}{1+\xi}<1\iff 1-\dfrac{\xi}{x}<1+\xi \iff -\dfrac{\xi}{x}<\xi \iff \xi\left(1+x\right) < 0 \iff \xi<0,\,x+1>0 \quad \checkmark \]

Riassumendo, abbiamo dimostrato che vale

    \[ 0 < \frac{1-\dfrac{\xi}{x}}{1+\xi} \le 1 \]

per ogni x \in (-1,1) \setminus \{0\}; ne segue che

    \[ \begin{aligned} \left \vert R_{n}(x;0) \right \vert & =\left \vert (-1)^{n}\Biggl{(}\alpha\prod_{j=1}^{n} \Bigl{(} 1-\frac{\alpha}{j} \Bigr{)} \Biggr{)}(1+\xi)^{\alpha-1}\left(\frac{1-\dfrac{\xi}{x}}{1+\xi}\right)^nx^{n+1}\right \vert  \\ &=\underbrace{\left \vert (-1)^{n}\right \vert}_{=1} \left \vert\alpha \right \vert\left \vert\prod_{j=1}^{n}\left(1-\dfrac{\alpha}{j}\right) \right \vert\underbrace{\left \vert \left(1+\xi\right)^{\alpha-1}\right \vert}_{\leq \left(1+\left \vert x \right\vert \right)^{\alpha-1}} \underbrace{\left \vert \left(\dfrac{1-\dfrac{\xi}{x}}{1+\xi}\right)^n\right \vert}_{\leq 1} \left \vert x^n\right \vert \left \vert x \right \vert   \\ &\leq \left \vert\alpha \right \vert \left \vert\underbrace{\left(1-\dfrac{\alpha}{1}\right)\left(1-\dfrac{\alpha}{2}\right)\dots\left(1-\dfrac{\alpha}{n}\right) }_{\text{$n$ termini}}\right \vert \left(1+\left \vert x \right \vert \right)^{\alpha -1}\left \vert \underbrace{x\cdot x\dots x}_{\text{$n$ termini}}\right \vert \left \vert x \right \vert\\ &=\left \vert \alpha x  \right \vert \left(1+\left \vert x \right \vert\right)^{\alpha -1}\left \vert x \left(1-\dfrac{\alpha}{1}\right)x \left(1-\dfrac{\alpha}{2}\right)\dots x \left(1-\dfrac{\alpha}{n}\right) \right \vert\\  & = \left \vert \alpha x \right \vert\left(1+\left \vert x \right \vert\right)^{\alpha -1} \prod_{j=1}^{n}\left \vert x \right \vert \left \vert 1-\dfrac{\alpha }{j}\right \vert \end{aligned} \]

è verificata per ogni x \in \left(-1,1\right) ed ogni n \in \mathbb N \setminus \{1\}. Il lemma 2 ci dice che nel caso 0<|x|<1 esiste 0\leq q<1 tale che |x|<q, e dunque

    \[ \left \vert x\left(1-\dfrac{\alpha}{n}\right)\right \vert<q \]

per n>N, con N abbastanza grande. Manipolando in maniera opportuna il termine che conclude la stima sul resto poco sopra, vediamo subito che

    \[ \begin{aligned} \left \vert \alpha x \right \vert\left(1+\left \vert x \right \vert\right)^{\alpha -1} \prod_{j=1}^{n}\left \vert x \right \vert \left \vert 1-\dfrac{\alpha }{j}\right \vert & =\left \vert \alpha x \right \vert\left(1+\left \vert x \right \vert\right)^{\alpha -1}\underbrace{\left(\prod_{j=1}^{N}\left \vert x \right \vert \left \vert 1-\dfrac{\alpha }{j}\right \vert\right)}_{\text{$N$ elementi}}\underbrace{\left(\prod_{j=N+1}^{n}\left \vert x \right \vert \left \vert 1-\dfrac{\alpha }{j}\right \vert\right)}_{\text{$n-N$ elementi}}\\ &=\left \vert \alpha x \right \vert\left(1+\left \vert x \right \vert\right)^{\alpha -1}\left(\underbrace{\left \vert x \right \vert\left \vert 1-\dfrac{\alpha}{1}\right \vert}_{\leq \left(1+\left \vert \alpha \right \vert \right)} \dots \underbrace{\left \vert x \right \vert \left \vert 1-\dfrac{\alpha}{N}\right \vert }_{\leq \left(1+\left \vert \alpha \right \vert \right)}\right)\cdot \\ & \qquad \cdot \left(\underbrace{\left \vert x \right \vert \left \vert 1-\dfrac{\alpha}{N+1}\right \vert}_{\leq q}\dots\underbrace{ \left \vert x \right \vert \left \vert 1-\dfrac{\alpha}{n}\right \vert}_{\leq q} \right)\\ &< \left \vert \alpha x \right \vert \left(1+\left \vert x \right \vert \right)^{\alpha -1}\left(1+\left \vert \alpha \right \vert \right)^{N}q^{n-N}, \end{aligned} \]

ovvero

    \[ |R_{n}(x;0)| < |\alpha| (1+|\alpha|)^N q^{n-N}(1+|x|)^{\alpha-1}. \]

Il termine a destra tende a zero per n\to + \infty quando x \in (-1,1), quindi lo stesso è vero per il resto dal teorema del confronto. In particolare, la funzione f(x)=(1+x)^\alpha è sviluppabile in serie di Taylor centrata in x_0=0 con intervallo massimale di convergenza I_{max}=(-1,1).


Sviluppi di McLaurin noti per le funzioni elementari.

Presentiamo gli sviluppi di Taylor centrati in x_0=0 (serie di McLaurin) delle principali funzioni elementari3.

Funzioni razionali ed algebriche:

1. \dfrac{1}{1-x} = 1+x+x^2+x^3+\cdots+x^n + o(x^n)

 

2. (1+x)^a = 1+ax+\dfrac{a(a-1)}{2}x^2 + \dfrac{a(a-1)(a-2)}{6}x^3+\cdots+ \binom{a}{n}x^n + o(x^n)

 

3. \sqrt{1+x} = 1+\dfrac{1}{2}x -\dfrac{1}{8}x^2 +\dfrac{x^3}{16}+\cdots+ (-1)^{n-1} \dfrac{(2n-3)!!}{(2n)!!}x^{n} + o(x^{n+1})

 

Funzioni esponenziali e logaritmiche:

4. e^x = 1+x+\dfrac{x^2}{2!}+\dfrac{x^3}{3!}+\cdots+\dfrac{x^n}{n!} + o(x^{n})

 

5. a^x = 1+x\ln(a)+\dfrac{x^2}{2!}\ln^2(a)+\dfrac{x^3}{3!}\ln^3(a)+\cdots+\dfrac{x^n}{n!}\ln^n(a) + o(x^{n}), \quad a>0

 

6. \ln(1+x) = x-\dfrac{x^2}{2}+\dfrac{x^3}{3}-\dfrac{x^4}{4}+\cdots+(-1)^{n+1}\dfrac{x^n}{n} + o(x^n)

 

7. -\ln(1-x) = x+\dfrac{x^2}{2}+\dfrac{x^3}{3}+\dfrac{x^4}{4}+\cdots+\dfrac{x^{n+1}}{n+1}+o(x^{n+1})

 

Funzioni trigonometriche:

8. \sin x = x-\dfrac{x^3}{3!}+\dfrac{x^5}{5!}-\dfrac{x^7}{7!}+\cdots+\dfrac{(-1)^n}{(2n+1)!}x^{2n+1} + o(x^{2n+1})

 

9. \cos x = 1-\dfrac{x^2}{2!}+\dfrac{x^4}{4!}-\dfrac{x^6}{6!}+\cdots+\dfrac{(-1)^n}{(2n)!}x^{2n} + o(x^{2n})

 

10. \tan x = x+\dfrac{1}{3}x^3+\dfrac{2}{15}x^5+\dfrac{17}{315}x^7+\dfrac{62}{2835}x^9 + o(x^{9})

 

11. \sec x = 1+\dfrac{x^2}{2}+\dfrac{5}{24}x^4+\dfrac{61}{720}x^6+o(x^6)

 

12. \csc x = \dfrac{1}{x}+\dfrac{x}{6}+\dfrac{7}{360}x^3+\dfrac{31}{15120}x^5 + o(x^5)

 

13. \cot x = \dfrac{1}{x}-\dfrac{x}{3}-\dfrac{x^3}{45}-\dfrac{2}{945}x^5 + o(x^5)

 

Funzioni trigonometriche inverse:

14. \quad \arcsin x = x+\dfrac{x^3}{6}+\dfrac{3}{40}x^5+\cdots+\dfrac{(2n)!}{4^n(n!)^2(2n+1)}x^{2n+1}+ o(x^{2n+1})

 

15. \arccos x = \dfrac{\pi}{2} - \arcsin x = \dfrac{\pi}{2} -  x - \dfrac{x^3}{6} - \dfrac{3}{40}x^5 + \cdots + \dfrac{(2n)!}{4^n (n!)^2 (2n+1)} x^{2n+1}+ o(x^{2n+1})

 

16. \quad \arctan x = x-\dfrac{x^3}{3}+\dfrac{x^5}{5}-\dfrac{x^7}{7}+\cdots+(-1)^n\dfrac{x^{2n+1}}{2n+1} + o(x^{2n+1})

 

Funzioni iperboliche e loro inverse:

17. \sinh x = x+\dfrac{x^3}{3!}+\dfrac{x^5}{5!}+\dfrac{x^7}{7!}+\cdots+\dfrac{x^{2n+1}}{(2n+1)!}+o(x^{2n+1})

 

18. \cosh x = 1+\dfrac{x^2}{2!}+\dfrac{x^4}{4!}+\dfrac{x^6}{6!}+\cdots+\dfrac{x^{2n}}{(2n)!}+o(x^{2n})

 

19. \tanh x = x-\dfrac{1}{3}x^3+\dfrac{2}{15}x^5-\dfrac{17}{315}x^7+\dfrac{62}{2835}x^9 + o(x^{9})

 

20. \arcsinh x = x-\dfrac{x^3}{6}+\dfrac{3}{40}x^5+\cdots+(-1)^n\dfrac{(2n)!}{4^n(n!)^2(2n+1)}x^{2n+1}+ o(x^{2n+1})

 

21. \arctanh x = x+\dfrac{x^3}{3}+\dfrac{x^5}{5}+\dfrac{x^7}{7}+\cdots+\dfrac{x^{2n+1}}{2n+1} + o(x^{2n+1})

   


  1. Ricordiamo la definizione di doppio fattoriale di n:

        \[ n!! = \begin{cases}  2 \cdot 4 \cdot 6 \cdot (n-2) \cdot n, & \text{per } n \text{ pari} \\ 3 \cdot 5 \cdot 7 \cdot (n-2) \cdot n, & \text{per } n \text{ dispari} \end{cases} \]

    dove 0!!=1!!=1.


 
 

Conseguenze ed applicazioni del teorema di Taylor

Introduzione.

In questa sezione, concludiamo la dispensa presentando varie applicazioni del teorema di Taylor.

Condizioni sufficienti per l'esistenza di estremi relativi.

Data una funzione f :(a,b)\to \mathbb R due volte derivabile in (a,b), e dato un punto x_0 tale che f'(x_0)=0, una condizione sufficiente affinché f(x) abbiamo massimo (risp. minimo) in x_0 è che

    \[ f''(x_0)<0 \qquad \left(\text{risp. } f''(x_0)>0 \right). \]

Il teorema di Taylor permette di generalizzare questo risultato.

Proposizione 1. Sia f: (a,b) \to \mathbb{R} una funzione n-1 volte derivabile in (a,b). Supponiamo inoltre che f(x) sia derivabile n volte in x_0 \in (a,b) e che valga

    \[ f'(x_0)=f''(x_0)=\dots=f^{(n-1)}(x_0)=0 \quad \text{e} \quad f^{(n)}(x_0)\neq 0. \]

Se n è pari allora si ha:

    \[\quad\]

  • la funzione f(x) possiede un massimo relativo in x_0 se f^{(n)}(x_0)<0.
  •  

  • la funzione f(x) possiede un minimo relativo in x_0 se f^{(n)}(x_0)>0.

Se n è dispari si ha:

    \[\quad\]

  • la funzione f(x) è decrescente in un intorno di x_0 se f^{(n)}(x_0)<0.
  •  

  • la funzione f(x) è crescente in un intorno di x_0 se f^{(n)}(x_0)>0.

    \[\quad\]

Dimostrazione. Possiamo applicare il teorema 1. Dato che le prime n-1 derivate valutate in x_0 sono nulle, possiamo scrivere

    \[ 	f(x)=f(x_0)+ \frac{f^{(n)}(x_0)}{n!}(x-x_0)^{n} + o((x-x_0)^{n}). 	\]

Ricordando la notazione o(1) per una quantità infinitesima per x \to x_0, possiamo raccogliere (x-x_0)^n:

(23)   \begin{equation*} 	f(x)-f(x_0)=\frac{(x-x_0)^n}{n!} \left(  f^{(n)}(x_0) + o(1) \right). 	\end{equation*}

Poiché \lim_{x \to x_0} o(1)=0, esiste \delta>0 tale che f^{(n)}(x_0) + o(1) ha lo stesso segno di f^{(n)}(x_0) per |x-x_0|<\delta. Pertanto se n è pari e f^{(n)}(x_0)>0, il fattore in evidenza nella (23) è positivo e si ha

    \[ 	f(x)-f(x_0)>0 \qquad \text{per ogni $|x-x_0|<\delta$}, 	\]

ovvero x_0 è un punto di minimo relativo. Al contrario, se f^{(n)}(x_0)<0 abbiamo che

    \[ 	f(x)-f(x_0)<0 \qquad \text{per ogni }|x-x_0|<\delta, 	\]

dunque x_0 è un punto di massimo relativo. Nel caso n dispari, invece, se f^{(n)}(x_0)>0 deduciamo che

    \[ \begin{aligned} 	& f(x)-f(x_0)>0 \quad & \text{per } x \in (x_0, x_0 + \delta),  	\\ & f(x)-f(x_0)<0 \quad & \text{per } x \in (x_0-\delta, x_0), 	\end{aligned} \]

ovvero f(x) è crescente in un intorno di x_0. Si ragiona in modo analogo per f^{(n)}(x_0)<0.

Osservazione 12. Come osservato nell’esempio 1, esistono funzioni le cui derivate di ogni ordine si annullano in un dato punto. In tal caso, le ipotesi della proposizione 1 non sono soddisfatte e, di conseguenza, non si può dire nulla sulla natura del punto x_0.


Esercizi applicazioni formula di Taylor con il resto di Lagrange.

In questa sezione, proponiamo alcuni esercizi che possono essere risolti utilizzando la formula di Taylor e la teoria introdotta fino a questo punto.

Esercizio 1. Dimostrare che il numero di Nepero (e) è irrazionale.

    \[\quad\]

Soluzione. Scriviamo lo sviluppo di Mclaurin con il resto di Lagrange della funzione esponenziale:

(24)   \begin{equation*} 		e^x=\sum_{k=0}^{n}\dfrac{x^k}{k!}+\dfrac{x^{n+1}e^{\xi}}{\left(n+1\right)!}, 		\end{equation*}

dove \xi\in(0,x). Supponiamo per assurdo che e sia razionale, e denotiamo con p e q i due interi coprimi tali per cui si può scrivere e=p/q. Allora, prendendo x=1 in (24), si trova la relazione

    \[ 		e=\dfrac{p}{q}=\sum_{k=0}^{n}\dfrac{1}{k!}+\dfrac{e^{\xi}}{\left(n+1\right)!} 		\]

per qualche \xi\in(0,1). Dato n \in \mathbb N con n > q (in modo da avere n! / q \in \mathbb Z), moltiplichiamo l’uguaglianza per n!:

    \[ 		n!\dfrac{p}{q}=\sum_{k=0}^{n}\dfrac{n!}{k!}+\dfrac{n!\,e^{\xi}}{\left(n+1\right)!}. 		\]

La sommatoria è, per costruzione, un numero intero, ovvero

    \[ 		\sum_{k=0}^{n}\dfrac{n!}{k!}=\dfrac{n!}{1}+\cdots+\dfrac{n!}{n!}=c\in\mathbb{Z}, 		\]

mentre il termine di resto soddisfa la seguente proprietà:

    \[ 		\dfrac{n!\,e^{\xi}}{\left(n+1\right)!}=\dfrac{e^{\xi}}{\left(n+1\right)}=d\in (0,1)\,\,\text{se}\,\,n\geq 2. 		\]

Tuttavia, questo non è possibile perché il termine a sinistra (n! \cdot p/q) è un intero, perciò non si può ottenere come somma tra c \in \mathbb Z e d \in (0,1).

Esercizio 2. Sia f : (a,b) \to \mathbb R due volte derivabile in (a,b) e supponiamo che f^\prime(a)=f^\prime(b)=0. Mostrare che esiste almeno un punto c \in (a,b) tale che

    \[ 	\left \vert f^{\prime \prime}(c)\right \vert \geq \dfrac{4}{\left(b-a\right)^2}\left\vert f\left(b\right)-f\left(a\right)\right\vert . 	\]

    \[\quad\]

Soluzione. Poiché f é derivabile due volte in (a,\dfrac{a+b}{2}) e in ( \dfrac{a+b}{2},b), possiamo svilupparla in serie di Taylor con il resto di Lagrange in entrambi gli intervalli separatamente:

(25)   \begin{equation*} 		\exists \zeta\in \left(a,\frac{a+b}{2}\right) \: : \: f\left( \frac{a+b}{2} \right)=f(a)+f^{\prime}(a)\left(\frac{b-a}{2}\right)+\dfrac{1}{2}f^{\prime\prime}\left(\zeta\right)\left(\frac{b-a}{2}\right)^2 		\end{equation*}

e, analogamente,

(26)   \begin{equation*} 		\exists \eta\in \left( \frac{a+b}{2},b\right) \: : \: f\left(  \frac{a+b}{2}\right)=f(b)+f^{\prime}(b)\left(\frac{a-b}{2}\right)+\dfrac{1}{2}f^{\prime\prime}\left(\eta\right)\left(\frac{a-b}{2}\right)^2. 		\end{equation*}

Sottraendo (25) e (26) e ricordando che f^\prime(a)=f^\prime(b)=0, otteniamo

    \[ 			f(b)-f(a)=\dfrac{1}{2}\left(f^{\prime\prime}\left(\zeta\right)\left( \frac{b-a}{2} \right)^2-f^{\prime \prime}\left(\eta\right)\left(\frac{a-b}{2}\right)^2\right). 		\]

D’altra parte, i due quadrati coincidono quindi possiamo stimare il termine a sinistra come segue:

    \[ \begin{aligned} 		|f(b)-f(a)| & =\left| \dfrac{1}{2}\left(f^{\prime\prime}\left(\zeta\right)\left( \frac{b-a}{2} \right)^2-f^{\prime \prime}\left(\eta\right)\left(\frac{a-b}{2}\right)^2\right) \right|  		\\ & = \dfrac{1}{2} \left( \frac{b-a}{2} \right)^2 \left| f^{\prime\prime}\left(\zeta\right) - f^{\prime \prime}\left(\eta\right) \right| 		\\ & \le \dfrac{1}{2} \left( \frac{b-a}{2} \right)^2 \left(\left \vert f^{\prime\prime}\left(\zeta\right)\right \vert + \left \vert f^{\prime \prime}\left(\eta\right)\right \vert \right) 		\\ & \le 2\cdot \dfrac{\left(b-a\right)^2}{8}\max\{\left \vert f^{\prime \prime }\left(\zeta\right)\right \vert ,\left \vert f^{\prime \prime }\left(\eta\right)\right \vert\} 		\\ & = \dfrac{\left(b-a\right)^2}{4}\max\{\left \vert f^{\prime \prime }\left(\zeta\right)\right \vert ,\left \vert f^{\prime \prime }\left(\eta\right)\right \vert\},         \end{aligned}\]

e questo conclude la dimostrazione.


Limiti con l'espansione in serie di Taylor.

Nel calcolo dei limiti, le forme indeterminate rappresentano una sfida particolare che richiede un approccio metodico e ragionato. Per risolverle, possiamo ricorrere a diverse strategie: dalle semplici manipolazioni algebriche all’applicazione del teorema di de l’Höpital, fino all’utilizzo dei limiti notevoli e degli sviluppi di Taylor-McLaurin. Se le prime tecniche risultano generalmente più familiari agli studenti, l’impiego dei limiti notevoli e degli sviluppi in serie spesso genera incertezze. Questo ci porta naturalmente a porci alcune domande fondamentali:

    \[\quad\]

  1. Quando è preferibile utilizzare i limiti notevoli rispetto agli sviluppi di Taylor?
  2.  

  3. Come determinare l’ordine appropriato dello sviluppo di Taylor?
  4.  

  5. Come gestire correttamente gli o-piccolo?
  6.  

  7. Qual è la connessione tra limiti notevoli e sviluppi di Taylor?

In questa sezione affronteremo queste questioni attraverso esempi pratici, illustrando il processo decisionale che guida la scelta del metodo più efficace per ogni tipo di problema.

Esercizio 3. Calcolare il seguente limite:

    \[ \lim_{x\to 0} \dfrac{\sin x - x}{3x^3}. \]

    \[\quad\]

Soluzione. Il lettore sarà probabilmente familiare con il limite notevole del seno

    \[   \lim_{x\to 0} \frac{\sin x}{x}=1.   \]

Tuttavia, se proviamo ad utilizzarlo, arriviamo ad una forma indeterminata:

    \[ \lim_{x\to 0} \frac{ \sin x - x}{3x^3}=\lim_{x\to 0} \frac{ x \left(\dfrac{\sin x}{x} - 1\right)}{3x^3}= \lim_{x\to 0} \frac{ \left(\dfrac{\sin x}{x} - 1\right)}{3x^2}=\dfrac{0}{0}, \]

pertanto non possiamo concludere. Sviluppiamo perciò il seno all’ordine successivo, che ricordiamo essere 3 perché solo i termini dispari sono non-nulli; si ha

    \[ \sin(x) = x - \frac{x^3}{6} + o(x^3), \]

da cui si trova il risultato:

    \[ \lim_{x\to 0} \frac{\sin x - x}{3x^3}=\lim_{x\to 0} \frac{x -\dfrac{x^3}{6} +o(x^3)- x}{3x^3}=\lim_{x\to 0} \frac{ -\dfrac{1}{6} +o(1)}{3}=-\frac{1}{18}. \]

Esercizio 4. Calcolare il seguente limite:

    \[ \lim_{x\to 0} \frac{e^x - 1}{\ln(1+x)}. \]

    \[\quad\]

Soluzione. Questo limite può essere risolto in due modi distinti. Vediamoli entrambi per comprendere quale approccio sia più efficiente.

Metodo 1: Sviluppi di Taylor Sviluppando sia l’esponenziale che il logaritmo al primo ordine4, otteniamo:

    \[\begin{aligned} \lim_{x\to 0} \frac{e^x - 1}{\ln(1+x)} &= \lim_{x\to 0} \frac{1+x +o(x)- 1}{x+o(x)} \\ &= \lim_{x\to 0} \frac{x(1 +o(1))}{x(1+o(1))} = 1. \end{aligned}\]

Metodo 2: Limiti Notevoli Un approccio più elegante sfrutta i limiti notevoli fondamentali:

    \[\begin{aligned} \lim_{x\to 0} \frac{e^x - 1}{\ln(1+x)} &= \lim_{x\to 0} \left(\frac{e^x-1}{x}\right) \cdot \left(\frac{x}{\ln(1+x)}\right) \\ &= \lim_{x\to 0} \frac{e^x-1}{x} \cdot \lim_{x\to 0} \frac{x}{\ln(1+x)} \\ &= 1 \cdot 1 = 1, \end{aligned}\]

dove abbiamo utilizzato i limiti notevoli \displaystyle \lim_{x\to 0} \frac{e^x - 1}{x}=1 e \displaystyle \lim_{x\to 0} \frac{\ln(1+x)}{x}=1.

Osservazione 13. Il secondo metodo risulta più immediato e elegante, poiché sfrutta direttamente i limiti notevoli senza richiedere manipolazioni degli sviluppi di Taylor. Questo esempio mostra come, in alcuni casi, l’utilizzo dei limiti notevoli possa offrire una strada più diretta rispetto agli sviluppi in serie.

Ci si potrebbe chiedere perché, nell’esercizio 3, il limite notevole del seno non sia stato sufficiente per risolvere il problema. La risposta risiede in un concetto fondamentale degli sviluppi in serie: l’ordine di approssimazione.

Mentre i limiti notevoli considerano essenzialmente il comportamento al primo ordine delle funzioni, in molti casi è necessario esaminare anche i termini di ordine superiore. Infatti, gli infinitesimi di ordine maggiore, che vengono trascurati nei limiti notevoli, possono essere determinanti per il calcolo del limite. Questo ci conduce a una questione cruciale nella teoria degli sviluppi di Taylor: come determinare l’ordine appropriato di sviluppo per risolvere un dato problema.

Determinazione dell’Ordine di Sviluppo

La scelta dell’ordine appropriato nello sviluppo di Taylor richiede un’attenta analisi del problema e spesso si affina con l’esperienza. Tuttavia, possiamo individuare alcune linee guida fondamentali da cui partire:

    \[\quad\]

  1. Nel caso di somme di funzioni, è necessario sviluppare fino al primo ordine che produce un risultato non nullo. Come visto nell’esercizio 3, quando il primo ordine fornisce un risultato nullo, occorre procedere con gli ordini successivi (nel caso specifico, il terzo per la funzione seno).
  2.  

  3. Nel caso in cui ci sia il rapporto tra una funzione ed un polinomio, generalmente è necessario sviluppare la prima fino al grado del polinomio presente nell’espressione.

La scelta dell’ordine corretto può essere validata attraverso un’analisi attenta del resto di Peano. Torniamo ad analizzare l’esercizio 3 per illustrare questo concetto nella pratica:

    \[\quad\]

  1. Sviluppando al primo ordine:

        \begin{equation*}         \lim_{x\to 0} \frac{x +o(x)- x}{3x^3}=\lim_{x\to 0} \frac{o(x)}{x^3}     \end{equation*}

    L’indeterminazione del rapporto \dfrac{o(x)}{x^3} indica che l’ordine di sviluppo è insufficiente.

  2.  

  3. Sviluppando fino al quinto ordine:

        \[\begin{aligned}         \lim_{x\to 0} &\frac{x -\dfrac{x^3}{6}+\dfrac{x^5}{5!}+o(x^5)- x}{3x^3} \\         &= \lim_{x\to 0} \left(-\frac{1}{18}+\frac{x^2}{3\cdot 5!}+\frac{o(x^5)}{3x^3}\right) \\         &= -\frac{1}{18}+\lim_{x \to 0}\frac{x^2}{3\cdot5!}+\lim_{x \to 0}o(x^2) \\         &= -\frac{1}{18}+0+0 = -\frac{1}{18}     \end{aligned}\]

Per concludere, facciamo qualche osservazione su i due risultati appena ottenuti:

    \[\quad\]

  • In questo caso specifico, lo sviluppo al terzo ordine sarebbe stato sufficiente5.
  •  

  • È preferibile, in caso di dubbio, sviluppare a un ordine superiore: si può sempre semplificare in seguito i termini non necessari. Ovviamente, va tenuta in conto la difficoltà nel calcolo di derivate di ordini superiori se non si sta utilizzando uno sviluppo in serie già noto.
  •  

  • Per completare la padronanza degli sviluppi di Taylor nel calcolo dei limiti, è fondamentale saper manipolare correttamente gli o-piccoli, utilizzando l’algebra degli infinitesimi data nella definizione 1.

Esercizio 5. Calcolare il seguente limite:

    \begin{equation*} \lim_{x\to+\infty}  x^{2x} \left( e^{\frac{1}{x}\ln \left(1+\frac{1}{x}\right)}-1\right)^x. \end{equation*}

    \[\quad\]

Soluzione. Procediamo per passi, semplificando questo limite apparentemente complesso.

Passo 1: Cambio di variabile. Poiché il limite è per x \to +\infty, effettuiamo il cambio di variabile t=\dfrac{1}{x}, che trasforma il limite in:

(27)   \begin{equation*} \lim_{t\to0^+} \left(\frac{1}{t}\right)^{\frac{2}{t}} \left(e^{t\ln(1+t)}-1\right)^{\frac{1}{t}} = \lim_{t\to0^+} \left(\frac{e^{t\ln(1+t)}-1}{t^2}\right)^{\frac{1}{t}}. \end{equation*}

Questa scrittura ha un vantaggio ulteriore: essendo il limite per t \to 0^+, possiamo facilmente sfruttare gli sviluppi delle funzioni note viste nelle sezioni precedenti.

Passo 2: Sviluppo in serie Taylor. Dobbiamo sviluppare l’espressione e^{t\ln(1+t)}. Trattandosi di una funzione composta, partiamo dalla funzione più interna verso la più esterna:

    \[\quad\]

  1. Sviluppo del logaritmo:

        \[\ln(1+t) = t-\frac{t^2}{2} + o(t^2)\]

    La scelta dell’ordine 2 è suggerita dalla presenza di t^2 al denominatore in (27).

  2.  

  3. Composizione con l’esponenziale:

        \[e^{t\ln(1+t)} = e^{t^2-\frac{t^3}{2}+o(t^3)}\]

  4.  

  5. Ricordando che e^x = 1 + x + x^2 +o(x^2), con x=t^2-t^3/2 + o(t^3), possiamo fermare lo sviluppo al secondo ordine (e non al primo ordine, come potrebbe sembrare sensato – vedi Osservazione 14):

    (28)   \begin{equation*}     e^{t^2-\frac{t^3}{2}+o(t^3)} = 1+t^2-\frac{t^3}{2}+o(t^3)     \end{equation*}

    I termini di ordine superiore, ovvero quelli relativi ad x^2, vengono inglobati in o(t^3).

Passo 3: Semplificazione del limite. Sostituendo (28) in (27) si ottiene

    \[\begin{aligned}     \lim_{t\to0} \left(\frac{1+t^2-\dfrac{t^3}{2}+o(t^3)-1}{t^2}\right)^{\frac{1}{t}}     &= \lim_{t\to0} \left(1-\frac{t}{2}+o(t)\right)^{\frac{1}{t}}. \end{aligned}\]

Passo 4: Risoluzione della forma indeterminata [1]^{[\infty]}. Per risolvere questa forma indeterminata, utilizziamo la proprietà

    \[ f(t)^{g(t)} = e^{g(t)\ln f(t)} \]

valida per f(t)>0. Si trova

(29)   \begin{equation*} \lim_{t\to0^+} \left(1-\frac{t}{2}+o(t)\right)^{\frac{1}{t}} = e^{\displaystyle\lim_{t\to 0^+}\frac{1}{t}\ln\left(1-\frac{t}{2}+o(t)\right)} \end{equation*}

Passo 5: Calcolo del limite. L’ultimo limite si risolve osservando che

    \[ \begin{aligned} \lim_{t\to 0^+}\frac{\ln\left(1-\dfrac{t}{2}+o(t)\right)}{t} &= \lim_{t\to 0^+}\frac{-\dfrac{t}{2}+o(t)}{t} \\ &= - 1/2, \end{aligned}\]

da cui segue che

Osservazione 14. (importanza dell’Ordine di Sviluppo). Analizziamo cosa accade quando si sceglie un ordine di sviluppo inadeguato, evidenziando come una scelta errata possa compromettere l’intera soluzione. Ad esempio, supponiamo di arrestare lo sviluppo dell’esponenziale al primo ordine:

    \[ e^x = 1 + x + o(x). \]

Come accennato in precedenza, questa scelta potrebbe sembrare ragionevole. In effetti, i termini derivanti da x^2 vengono inglobati in o(t^3); tuttavia, il vero problema è la presenza del termine o(x) invece di o(x^2):

(30)   \begin{equation*} e^{t^2-\frac{t^3}{2}+o(t^3)}=1+\left(t^2-\frac{t^3}{2}+o(t^3)\right) + o\left(t^2-\frac{t^3}{2}+o(t^3)\right). \end{equation*}

Questo sviluppo porta a due problemi fondamentali:

    \[\quad\]

  1. Il termine o\left(t^2-\dfrac{t^3}{2}+o(t^3)\right) si riduce a o(t^2), modificando lo sviluppo in:

        \begin{equation*}     e^{t^2-\frac{t^3}{2}+o(t^3)}=1+t^2 + o(t^2),     \end{equation*}

    dunque viene a mancare il termine di grado 3.

  2.  

  3. Proseguendo con questa espressione, il limite diventa:

        \[\begin{aligned}     \lim_{t\to0^+} \left(\frac{e^{t\ln(1+t)}-1}{t^2}\right)^{\frac{1}{t}}      &= \lim_{t\to0^+} \left(\frac{1+t^2 + o(t^2)-1}{t^2}\right)^{\frac{1}{t}} \\     &= \lim_{t\to0^+} (1+o(1))^{\frac{1}{t}} \\     &= \lim_{t\to0^+} e^{\frac{1}{t}\ln(1+o(1))} \\     &= \lim_{t\to0^+} e^{\frac{1}{t}(o(1))}     \end{aligned}\]

Il limite si riduce alla conoscenza di \lim_{t\to0}\dfrac{o(1)}{t}, una forma indeterminata che non può essere risolta senza ulteriori informazioni sulla natura precisa dell’o-piccolo. Questo dimostra come la scelta dell’ordine di sviluppo sia cruciale per la corretta risoluzione del problema.

Esercizio 6. Calcolare il seguente limite, se esiste:

    \[ \lim_{x\to0^+} \frac{(1+x-\dfrac{x^3}{6}-\sin(x))^{\frac{5}{x}}-1}{\log(1+x^4)} \]

    \[\quad\]

Soluzione. Iniziamo con lo sviluppare in serie di Taylor il seno e il logaritmo intorno ad x_0 = 0; si ha

    \[ \sin(x) = x-\frac{x^3}{6}+\frac{x^5}{120}+o(x^5), \qquad \log(1+x) = x+o(x). \]

Lo sviluppo del seno al quinto ordine è dovuto al fatto che gli altri termini si elidono con il polinomio presente a numeratore. In particolare, il termine elevato alla 5/x è dato da

    \[ 1+x-\frac{x^3}{6}-\sin(x) = 1-\frac{x^5}{120}+o(x^5). \]

A questo punto, ricordando che logaritmo ed esponenziale sono una la funzione inversa dell’altra, otteniamo

    \[ \left(1-\frac{x^5}{120}+o(x^5)\right)^{\frac{5}{x}} =  \exp\left[\frac{5}{x}\log\left(1-\frac{x^5}{120}+o(x^5)\right)\right] = \exp\left(-\frac{x^4}{24}+o(x^4)\right) \]

Infine, consideriamo lo sviluppo al primo ordine dell’esponenziale (e^x = 1+x+o(x)) e scriviamo il numeratore come segue:

    \[ (1+x-\frac{x^3}{6}-\sin(x))^{\frac{5}{x}}-1 = \left(1 - \frac{x^4}{24} + o(x^4)\right) - 1 = - \frac{x^4}{24} + o(x^4). \]

Per il denominatore, utilizziamo lo sviluppo del logaritmo al primo ordine sostituendo x con x^4 in quanto composizione tra una funzione ed un polinomio. Ne segue che

    \[ \lim_{x\to0^+} \frac{(1+x-\dfrac{x^3}{6}-\sin(x))^{\frac{5}{x}}-1}{\log(1+x^4)} =  \lim_{x\to0^+} \frac{-\dfrac{x^4}{24}+o(x^4)}{x^4+o(x^4)} = -\frac{1}{24}, \]

concludendo così l’esercizio.

   


  1. Usiamo gli sviluppi noti per x\to 0: e^x=1+x+o(x) e \ln (1+x)=x+o(x).
  2.  

    1. Ricordiamo che per funzioni dispari lo sviluppo di Taylor centrato in x_0=0 contiene solo termini di grado dispari, mentre per funzioni pari solo termini di grado pari.

Approssimazioni di numeri trascendenti.

Presentiamo ora alcuni esempi d’applicazione dello sviluppo di Taylor all’approssimazione di numeri trascendenti6.

Esempio 2 (il numero di Nepero). La serie di Taylor ci permette di approssimare il numero e con precisione arbitraria e dimostrarne l’irrazionalità. Utilizzando l’espressione del resto in forma di Lagrange per la funzione esponenziale con x=1 (e quindi |\xi|\le 1), otteniamo:

(31)   \begin{equation*} |R_n(x=1;0)| \le \frac{\mathrm{e}}{(n+1)!} \end{equation*}

Per ottenere \beta cifre decimali esatte, è sufficiente che (n+1)!>\mathrm{e}\cdot10^\beta. Ad esempio, per 5 cifre significative bastano i primi 8 termini della serie dato che

    \[ 9! =362.880 > 271.828,183 = e \cdot 10^5. \]

L’irrazionalità di e può essere dimostrata per assurdo. Supponiamo di poter scrivere e=\dfrac{p}{q} con p,q \in \mathbb{N} numeri interi coprimi e q \neq 1. Usando l’espansione fino al termine q, si ha

    \[ |R_q(x=1;0)|=\Bigl{|}\mathrm{e}- \sum_{k=0}^q \frac{1}{k!} \Bigr{|}=\Bigl{|}\frac{p}{q} - \sum_{k=0}^q \frac{1}{k!} \Bigr{|} \le \frac{\mathrm{e}}{(q+1)!} \]

Moltiplicando per q! a destra e sinistra otteniamo:

    \[ \left| p(q-1)!-\sum_{k=0}^q \frac{q!}{k!} \right| < \frac{3}{q+1}. \]

Tuttavia, il termine a sinistra è un intero positivo (per cui il modulo si può togliere), mentre il termine a destra è minore o uguale di uno; questo non è possibile, concludendo la dimostrazione.

Esempio 3 (tavole logaritmiche). Sostituendo x \to -x nella (19) otteniamo:

(32)   \begin{equation*} -\log(1-x)=\sum_{k=0}^n \frac{x^{k+1}}{k+1} +\int_0^{x} \frac{s^{n+1}}{1-s} \, ds \end{equation*}

Sommando (32) e (19) moltiplicate per una opportuna costante (i.e., 1/2) ed n=2m si ricava l’identità:

(33)   \begin{equation*} \frac{1}{2} \log{\frac{1+x}{1-x}}=\sum_{k=0}^{m} \frac{x^{2k+1}}{2k+1} + \int_0^x \frac{s^{n+1}}{1-s^2} \, ds. \end{equation*}

Si noti che i termini di grado pari nello sviluppo in serie di Taylor sono nulli. Inoltre, il resto in forma integrale è dato da quello scritto in (33) solo per n pari perché

    \[ \int_0^x (-1)^{n+1} \frac{s^{n+1}}{1+s} \, ds + \int_0^x \frac{s^{n+1}}{1-s} \, ds = \int_0^x \frac{s^n}{1-s^2} \, ds \iff n = 2m. \]

Per -1<x<1, l’immagine della funzione x \to \frac{1+x}{1-x} è l’insieme dei reali positivi, permettendo così il calcolo del logaritmo naturale di qualsiasi numero positivo.

Esempio 4 (la costante di Archimede e le formule di Machin). Lo sviluppo di Taylor dell’arcotangente (20) per x=1 fornisce la serie comunemente nota come quella di Leibnitz-Gregory:

    \[ \frac{\pi}{4}= \arctan(1)=1- \frac{1}{3} + \frac{1}{5} - \frac{1}{7} + \dots= \sum_{k=0}^\infty (-1)^k \frac{x^{2k+1}}{2k+1}. \]

Questa serie converge molto lentamente: per ottenere \beta cifre decimali esatte serve n>5 \cdot 10^{\beta -1} termini, rendendo il calcolo impraticabile per approssimazioni precise di \pi. Ad esempio, per avere \beta=10 cifre decimali esatte, i termini da sommare sono più di 5 miliardi.

La formula di Machin offre una soluzione più efficiente. Definendo \alpha=\arctan(1/5) e utilizzando le formule di duplicazione per la tangente, ovvero

    \[ \tan(\delta+\gamma)=\frac{\tan \delta + \tan \gamma}{1- \tan \delta \tan \gamma}, \]

possiamo scrivere \pi/4 come segue:

(34)   \begin{equation*} \frac{\pi}{4}=4\arctan\frac{1}{5}-\arctan\frac{1}{239}. \end{equation*}

Questa formula, combinata con lo sviluppo di Taylor, fornisce un’approssimazione molto più rapida. Il resto può essere stimato come segue:

    \[ 4\Bigl{|} R_{2n+1}\Bigl{(}\frac{1}{5};0\Bigr{)}\Bigr{|}+\Bigl{|} R_{2n+1}\Bigl{(}\frac{1}{239};0\Bigr{)}\Bigr{|}\le \frac{1}{2n+1}\Bigl{(} \frac{16}{5^{2n+1}}+\frac{1}{239^{2n+1}}\Bigr{)}<\frac{1}{(2n+1)5^{2n}}. \]

In questo caso, è facile verificare che per ottenere 10 cifre significative è sufficiente n>5, un miglioramento netto rispetto alla serie di Leibnitz-Gregory menzionata in precedenza.    


  1. Ricordiamo che un numero trascendente \alpha è un numero che non è algebrico ovvero che non è uno zero di un polinomio a coefficienti razionali Q(x) \in \mathbb{Q}[x]. Sono numeri trascendenti e, \pi, \log(\alpha) con \alpha razionale positivo tale che \alpha\neq1.

 
 

Esercizi misti

    \[\quad\]

Esercizio 7. Calcolare, se esiste, il seguente limite di funzione:

    \[  \lim_{x\to0}\dfrac{\sin (x^4+x^2)-\arctan^2 (x^4+x)}{\sqrt{x^3+x^2} \ln (1+\sinh^3x)}. \]

Soluzione.

Calcoliamo questo limite analizzando separatamente numeratore e denominatore attraverso gli sviluppi di Taylor. Partiamo dal numeratore. Per x \to 0 abbiamo:

    \[\quad\]

  • Il termine \sin(x^4+x^2) si può sviluppare come funzione composta tra \sin t e un polinomio di grado 4:

        \[ \sin(x^4+x^2) = x^4+x^2+o(x^4). \]

  •  

  • Il termine \arctan^2(x^4+x) si ottiene sviluppando prima l’arcotangente di x^4+x e poi se ne prende il quadrato. Nello specifico, si ha

    (35)   \begin{equation*}    \arctan(x^4+x) = x-\frac{1}{3}x^3+o(x^3),    \end{equation*}

    da cui segue che

        \[ \arctan^2(x^4+x^2) = \left(x-\frac{1}{3}x^3+o(x^3)\right)^2 = x^2-\frac{2}{3}x^4+o(x^4). \]

A questo punto, ci possiamo occupare del denominatore per x \to 0:

    \[\quad\]

  • La radice si sviluppa come:

        \[ \sqrt{x^3+x^2} = x(1+o(1)) \]

  •  

  • Per il logaritmo, usando lo sviluppo del seno iperbolico (vedi la sezione precedente):

        \[ \ln(1+\sinh^3(x)) = \ln(1+x^3+o(x^3)) = x^3+o(x^3). \]

Combinando tutti gli sviluppi7:

    \[\begin{aligned} \lim_{x\to0}\frac{\sin(x^4+x^2)-\arctan^2(x^4+x)}{\sqrt{x^3+x^2}\ln(1+\sinh^3x)} &= \lim_{x\to0}\frac{x^4+\cancel{x^2}-\cancel{x^2}+\dfrac{2}{3}x^4+o(x^4)}{x(1+o(1))(x^3+o(x^3))} \\ &= \lim_{x\to0}\frac{\dfrac{5}{3}x^4+o(x^4)}{x(1+o(1))(x^3+o(x^3))} = \frac{5}{3}. \end{aligned}\]

   


  1. Il limite esiste finito poiché i limiti per x\to0^- e per x\to0^+ esistono e coincidono.

 
 

Esercizio 8. Calcolare, se esiste, il seguente limite di funzione:

    \[ \lim_{x\to0} \frac{\sin\left(\dfrac{1}{2x+1}-\dfrac{1}{e^{2x}}\right)}{\ln(\cosh(3x))} \]

Soluzione.

Analizziamo ancora una volta numeratore e denominatore separatamente, utilizzando gli sviluppi di Taylor. Per il numeratore, dobbiamo prima sviluppare l’argomento del seno. Per x\to0:

    \[\quad\]

  • Sviluppo di \dfrac{1}{2x+1}:

        \[ \frac{1}{2x+1} = 1-2x+4x^2+o(x^2) \]

  •  

  • Sviluppo di -\dfrac{1}{e^{2x}}:

        \[ -\frac{1}{e^{2x}} = -e^{-2x} = -1+2x+2x^2-4x^2+o(x^2) \]

Si noti che la scelta dell’ordine di sviluppo viene fatta, in un certo senso, a posteriori. Infatti, l’argomento del seno non deve essere identicamente nullo e questo richiede di sviluppare all’ordine 2. Infatti, si ha:

    \[\begin{aligned} \frac{1}{2x+1}-\frac{1}{e^{2x}} &= (1-2x+4x^2+o(x^2))+(-1+2x+2x^2-4x^2+o(x^2)) \\ &= 2x^2+o(x^2). \end{aligned}\]

Ricordando che \sin t = t + o(t), prendendo t = 2x^2 + o(x^2), otteniamo il seguente sviluppo per il numeratore:

    \[ \sin\left(\frac{1}{2x+1}-\frac{1}{e^{2x}}\right) = \sin(2x^2+o(x^2)) = 2x^2+o(x^2) \]

Facendo riferimento alla sezione precedente per lo sviluppo del coseno iperbolico, il denominatore è dato da

    \[ \ln(\cosh(3x)) = \ln\left(1+\frac{9}{2}x^2+o(x^2)\right) = \frac{9}{2}x^2+o(x^2),\]

perciò combinando numeratore e denominatore si conclude l’esercizio:

    \[ \lim_{x\to0} \frac{\sin\left(\dfrac{1}{2x+1}-\dfrac{1}{e^{2x}}\right)}{\ln(\cosh(3x))}  = \lim_{x\to0} \frac{2x^2+o(x^2)}{\dfrac{9}{2}x^2+o(x^2)} = \frac{4}{9}. \]


 
 

Esercizio 9. Calcolare il seguente limite, se esiste:

    \[\displaystyle \lim_{x\to0^+} \dfrac{e\left[ 	\cos (\sqrt{2x}) \right]^{\frac{1}{x}}-1}{x}.\]

Soluzione.

Per x\to0^+ si ha

    \[ \begin{aligned}  \dfrac{e\left[ 	\cos (\sqrt{2x}) \right]^{\frac{1}{x}}-1}{x} & = \left[\cos (\sqrt{2x}) \right]^{\frac{1}{x}}= e^{\frac{1}{x}\ln\left(1- \frac{2x}{2}+\frac{2^2 x^2}{4!}+o(x^2) \right)}=e^{\frac{1}{x}\ln \left(1-x+\frac{x^2}{6}+o(x^2) \right)}\\ &=e^{\frac{1}{x} \left(-x+\frac{x^2}{6}-\frac{x^2}{2}+o(x^2) \right)}=e^{-1}e^{-\frac{x}{3}+o(x)} \\ &=e^{-1}\left(1-\dfrac{1}{3}x+o(x)\right) \end{aligned} \]

da cui

    \[ \begin{aligned} \displaystyle \lim_{x\to0^+} \dfrac{e\left[ 	\cos (\sqrt{2x}) \right]^{\frac{1}{x}}-1}{x}&=\lim_{x\to0^+}\dfrac{e\cdot e^{-1}\left(1-\dfrac{1}{3}x+o(x)\right)-1}{x}\\ &=\lim_{x\to0^+}\dfrac{-\dfrac{1}{3}x+o(x)}{x}=-\dfrac{1}{3}. \end{aligned} \]


 
 

Esercizio 10. Calcolare, se esiste, il seguente limite di funzione:

    \[ \lim_{x\to+\infty} \left( \frac{4^{1/x^2}+6^{1/x^2}}{3^{1/x^2}+5^{1/x^2}} \right)^{x^2}. \]

Soluzione (primo metodo).

Il limite presenta una forma indeterminata del tipo [1^\infty], poiché per x\to+\infty si ha 1/x^2 \to 0 e sia numeratore che denominatore tendono a 2. Procediamo a manipolare l’espressione. Procediamo con la seguente manipolazione:

    \[\begin{aligned}  \lim_{x\to+\infty} \left(\frac{4^{1/x^2}+6^{1/x^2}}{3^{1/x^2}+5^{1/x^2}}\right)^{x^2}&= \lim_{t\to0^+} \left(\frac{4^t+6^t}{3^t+5^t}\right)^{1/t} \\ &= \lim_{t\to0^+} \frac{6}{5}\left[\frac{1+(2/3)^t}{1+(3/5)^t}\right]^{1/t} \\ &= \lim_{t\to0^+} \frac{6}{5}\left[\frac{1+(2/3)^t+(3/5)^t-(3/5)^t}{1+(3/5)^t}\right]^{1/t}  \\ &= \lim_{t\to0^+} \frac{6}{5}\left[1+\frac{(2/3)^t-(3/5)^t}{1+(3/5)^t}\right]^{1/t} \\ &= \lim_{t\to0^+} \frac{6}{5}\left[1+\frac{(10/9)^t-1}{1+(3/5)^{-t}}\right]^{1/t}. \end{aligned}\]

Prima di continuare, ecco alcuni commenti sui passaggi:

    \[\quad\]

  1. Sostituzione t=1/x^2. Ovviamente x \to + \infty corrisponde a t \to 0^+.
  2.  

  3. Raccolta di 6^t e 5^t al numeratore e denominatore.
  4.  

  5. Aggiunta e sottrazione di (3/5)^t al numeratore.
  6.  

  7. Semplificazione tra numeratore e denominatore. Passaggio algebrico banale.
  8.  

  9. Moltiplicazione per (3/5)^{-t} a numeratore e denominatore.

Per concludere, utilizziamo il limite notevole:

(36)   \begin{equation*} \lim_{\varepsilon\to0}(1+\varepsilon)^{1/\varepsilon}=e. \end{equation*}

Nello specifico, utilizzando le proprietà delle potenze possiamo riscrivere il limite come

    \[\begin{aligned} \lim_{t\to0^+} \frac{6}{5}\left[1+\frac{(10/9)^t-1}{1+(3/5)^{-t}}\right]^{\frac{1+(3/5)^{-t}}{(10/9)^t-1}\cdot\frac{(10/9)^t-1}{1+(3/5)^{-t}}\cdot\frac{1}{t}} &= \frac{6}{5}\cdot e^{\frac{1}{2}\ln(10/9)} \\ &= \frac{6}{5}\sqrt{\frac{10}{9}} = \sqrt{\frac{8}{5}} \approx 1,265 \end{aligned}\]

dove abbiamo usato i seguenti limiti notevoli:

    \[ \frac{1}{1+(3/5)^{-t}}\to\frac{1}{2}, \qquad \frac{(10/9)^t-1}{t}\to\ln(10/9). \]


Soluzione (secondo metodo).

Procediamo utilizzato gli sviluppi di Taylor. Per ogni a > 0, vale:

(37)   \begin{equation*} a^t = 1+t\ln(a)+o(t) \qquad\text{per }t\to0. \end{equation*}

Mentre, per la serie geometrica, si ha

(38)   \begin{equation*} \frac{1}{1+t} = 1-t+o(t) \qquad\text{per }t\to0. \end{equation*}

Applicando questi sviluppi si arriva allo stesso risultato

    \[\begin{aligned} L &= \lim_{t\to0^+}\left(\frac{4^t+6^t}{3^t+5^t}\right)^{1/t} \\ &= \lim_{t\to0^+}\left(\frac{2+t\ln(24)+o(t)}{2+t\ln(15)+o(t)}\right)^{1/t} \\ &= \lim_{t\to0^+}\left(\frac{1+t\ln\sqrt{24}+o(t)}{1+t\ln\sqrt{15}+o(t)}\right)^{1/t} \\ &= \lim_{t\to0^+}\left[1+t\ln\sqrt{\frac{24}{15}}+o(t)\right]^{1/t} \\ &= \lim_{t\to0^+}\exp\left[\frac{1}{t}\ln\left(1+t\ln\sqrt{\frac{8}{5}}+o(t)\right)\right] \\ &= \sqrt{\frac{8}{5}}. \end{aligned}\]

Si noti che in questo caso abbiamo utilizzato L per indicare il limite. Questo è possibile perché nella prima soluzione abbiamo già dimostrato che il limite esiste ed è finito.


 
 

Esercizio 11. Si determinino a,b,c \in \mathbb{R} in modo che valga la seguente uguaglianza:

    \[ \lim_{x\to0}\frac{x(a+b\cos x)-c\arctan x}{x^5}=1. \]

Soluzione.

Risolviamo questo limite usando gli sviluppi di Taylor. Per x\to0 abbiamo:

    \[\quad\]

  • \cos x = 1-\dfrac{x^2}{2}+\dfrac{x^4}{24}+o(x^4)
  •  

  • \arctan x = x-\dfrac{x^3}{3}+\dfrac{x^5}{5}+o(x^5)

Sostituendo questi sviluppi nel numeratore ci porta a

    \[\begin{aligned} \frac{x(a+b\cos x)-c\arctan x}{x^5} &= \frac{x\left(a+b(1-\dfrac{x^2}{2}+\dfrac{x^4}{24})\right)-c(x-\dfrac{x^3}{3}+\dfrac{x^5}{5})+o(x^5)}{x^5} \\ &= \frac{x(a+b)-\dfrac{bx^3}{2}+\dfrac{bx^5}{24}-cx+\dfrac{cx^3}{3}-\dfrac{cx^5}{5}+o(x^5)}{x^5} \\ &= \underbrace{\frac{x(a+b-c)}{x^5}}_{=0}+\underbrace{\frac{x^3(-\dfrac{b}{2}+\dfrac{c}{3})}{x^5}}_{=0}+\underbrace{\frac{x^5(\dfrac{b}{24}-\dfrac{c}{5})}{x^5}}_{=1}. \end{aligned}\]

Perciò, affinché il limite sia uguale a 1, devono essere soddisfatte le seguenti condizioni:

    \begin{equation*} \begin{cases} a+b-c = 0 & \text{(termine che moltiplica }x \text{)} \\[1em] -\dfrac{b}{2}+\dfrac{c}{3} = 0 & \text{(termine che moltiplica }x^3\text{)} \\[1em] \dfrac{b}{24}-\dfrac{c}{5} = 1 & \text{(termine che moltiplica }x^5\text{)} \end{cases} \end{equation*}

Risolvendo il sistema8, si ottiene:

    \begin{equation*} \begin{cases} a = -\dfrac{60}{31} \\[1em] b = -\dfrac{120}{31} \\[1em] c = -\dfrac{180}{31} \end{cases} \end{equation*}

Questo risultato può essere verificato anche applicando il teorema di L’Hôpital cinque volte consecutive, ma è computazionalmente poco conveniente.

   


  1. I dettagli della risoluzione sono lasciati al lettore.

 
 

Esercizio 12. Calcolare, se esiste, il seguente limite di funzione:

    \[ \lim_{x \to +\infty} \left( \dfrac{2\arctan x}{\pi} \right)^{\pi x}. \]

Soluzione.

Iniziamo ricordandoci una importante identità trigonometrica:

    \[ 	\arctan x + \arctan\left( \dfrac{1}{x}\right)=\begin{cases} 	\,\,\,\,\,\dfrac{\pi}{2}\quad & \text{per}\,\, x>0,\\ 	\\ 	-\dfrac{\pi}{2}&\text{per}\,\, x<0. 	\end{cases} \]

Sostituendo nel limite, otteniamo

    \[ \lim_{x \to +\infty} \left( \dfrac{2\arctan x}{\pi} \right)^{\pi x} = \lim_{x \to +\infty} e^{\pi x \ln \left( \frac{2\arctan x}{\pi} \right)} = \lim_{x \to +\infty} e^{\pi x \ln \left( \frac{2\left(\frac{\pi}{2}-\arctan \frac{1}{x}\right)}{\pi} \right)}, \]

perciò possiamo concentrarci sul limite dell’esponente a destra. Dato che

    \[ \arctan\left(\dfrac{1}{x} \right)=\dfrac{1}{x}-\dfrac{1}{3}x^3+\dots+\left(-1 \right)^n\dfrac{x^{2n+1}}{2n+1}+o\left( x^{2n+1}\right) \]

per x \to + \infty, sostituendo possiamo scrivere:

    \[ \dfrac{\pi}{2}-\arctan\left( \dfrac{1}{x}\right)=\dfrac{\pi}{2}-\dfrac{1}{x}+o\left( \dfrac{1}{x}\right). \]

Ne segue che il limite è dato da

    \[ \begin{aligned}   \lim_{x \to +\infty} e^{\pi x \ln \left(\frac{2}{\pi} \left(\frac{\pi}{2}-\frac{1}{x}+o\left(\frac{1}{x}\right)\right)\right)}  &=\lim_{x \to +\infty} e^{\pi x\ln\left(1-\frac{2}{\pi x} + o\left(\frac{1}{x}\right)\right)} =\\ & =  \lim_{x \to +\infty} e^{\pi x \left(-\frac{2}{\pi x} + o\left(\frac{1}{x}\right)\right)} = e^{-2}, \end{aligned}   \]

concludendo così l’esercizio.


 
 

Esercizio 13. Calcolare, se esiste, il seguente limite di funzione:

(39)   \begin{equation*} \lim_{x\to0} |x|\left(\frac{e^{(\sin x)^2} +\tan x - \left(1+x+x^2+\frac{x^3}{3} + \frac{x^4}{6}\right)}{x^6}\right) \end{equation*}

Soluzione.

Per x\to0 abbiamo9:

    \[\begin{aligned} \tan x &= x+\frac{x^3}{3} + \frac{2}{15} x^5+ o(x^5) \\ \sin x &= x-\frac{x^3}{6} + o(x^3) \end{aligned}\]

Utilizzando quanto appena scritto, lo sviluppo di e^{(\sin x)^2} si ottiene per composizione:

    \[\begin{aligned} e^{(\sin x)^2} &= e^{(x-\frac{x^3}{6} + o(x^3))^2} \\ &= e^{(x^2 -\frac{x^4}{3}+o(x^4))} \end{aligned}\]

da cui, usando lo sviluppo dell’esponenziale e^x = 1+ x+ \dfrac{x^2}{2}+\dfrac{x^3}{6}+o(x^3), si ottiene:

    \[\begin{aligned} e^{(\sin x)^2} &= 1+\left(x^2 -\frac{x^4}{3}\right)+\frac{(x^2)^2}{2} + o(x^4) \\ &= 1+x^2+\frac{x^4}{6} + o(x^4). \end{aligned}\]

Sommando gli sviluppi di e^{(\sin x)^2} e \tan x:

    \[ e^{(\sin x)^2} +\tan x = 1+x+x^2+\frac{x^3}{3}+\frac{x^4}{6}+\frac{2}{15}x^5+o(x^5) \]

e sostituendo nell’espressione originale (39) otteniamo:

    \[ \lim_{x\to0} |x|\left(\frac{e^{(\sin x)^2} +\tan x - \left(1+x+x^2+\frac{x^3}{3} + \frac{x^4}{6}\right)}{x^6}\right) = \lim_{x\to0} |x|\left(\frac{\frac{2}{15}x^5 + o(x^5)}{x^6}\right). \]

A questo punto, data la presenza del modulo, calcoliamo i limiti destro e sinistro per verificare l’esistenza del limite per x \to 0. Da destra si ha

(40)   \begin{equation*} \lim_{x\to0^+} \frac{\dfrac{2}{15}x^6 + o(x^6)}{x^6} = \frac{2}{15}, \end{equation*}

mentre da sinistra

(41)   \begin{equation*} \lim_{x\to0^-} -\frac{\dfrac{2}{15}x^6 + o(x^6)}{x^6} = -\frac{2}{15}. \end{equation*}

Poiché i limiti laterali sono diversi10, concludiamo che il limite (39) non esiste.

   


  1. Sviluppiamo la tangente fino al quinto ordine poiché dovremo confrontarla con un polinomio di quarto grado
  2.  

    1. Per il teorema di unicità del limite, se un limite esiste deve essere unico

 
 

Esercizio 14. Calcolare, se esiste, il seguente limite di funzione:

    \[ \lim_{x\to 0^+} \frac{2\sqrt{1+\sin x\ln (x)}-\sin (x \ln (x))-2\cos\left(\frac{1}{2}x\ln x \right)}{2\tan(x) \ln (x)-\ln (1+2x \log(x))-2x^2 (\ln x)^2}. \]

Soluzione.

Risolviamo questo limite analizzando separatamente numeratore e denominatore attraverso gli sviluppi di Taylor. Nel primo caso, il numeratore, abbiamo:

    \[\quad\]

  1. Il termine 2\sqrt{1+\sin x\ln(x)} si sviluppa come segue:

        \[\begin{aligned} 2\sqrt{1+\sin x\ln(x)}&= 2\left(1+x\ln x-\frac{1}{6}x^3\ln x+o(x^3\ln x)\right)^{\frac{1}{2}} \\ &= 2\left(1+\frac{1}{2}x\ln x-\frac{1}{8}x^2(\ln x)^2+\frac{1}{16}x^3(\ln x)^3+o(x^3(\ln x)^3)\right) \\ &= 2+x\ln x-\frac{1}{4}x^2(\ln x)^2+\frac{1}{8}x^3(\ln x)^3+o(x^3(\ln x)^3). \end{aligned}\]

  2.  

  3. Gli altri termini, invece, si sviluppano come segue:

        \[\begin{aligned} -\sin(x\ln(x))-2\cos(\frac{1}{2}x\ln x) &= -x\ln x+\frac{1}{6}x^3(\ln x)^3-2\left(1-\frac{1}{8}x^2(\ln x)^2+o(x^3(\ln x)^3)\right) \\ &= -x\ln x+\frac{1}{6}x^3(\ln x)^3-2+\frac{1}{4}x^2(\ln x)^2+o(x^3(\ln x)^3). \end{aligned}\]

Combinando i risultati, il numeratore diventa:

(42)   \begin{equation*} \frac{7}{24}x^3(\ln x)^3+o(x^3(\ln x)^3). \end{equation*}

Siamo ora pronti a sviluppare il denominatore. Per x\to0^+, si ha

    \[\begin{aligned} &2\tan(x)\ln(x)-\ln(1+2x\ln(x))-2x^2(\ln x)^2 \\ &= 2x\ln x-\left(2x\ln x-\frac{1}{2}(4x^2)(\ln x)^2+\frac{1}{3}(8x^3)(\ln x)^3\right)-2x^2(\ln x)^2+o(x^3(\ln x)^3) \\ &= -\frac{8}{3}x^3(\ln x)^3+o(x^3(\ln x)^3). \end{aligned}\]

Per concludere, sostituiamo (42) e lo sviluppo del denominatore nel limite di partenza:

    \[\begin{aligned} \lim_{x\to0^+}&\frac{2\sqrt{1+\sin x\ln(x)}-\sin(x\ln(x))-2\cos(\frac{1}{2}x\ln x)}{2\tan(x)\ln(x)-\ln(1+2x\ln(x))-2x^2(\ln x)^2} \\ &= \lim_{x\to0^+}\frac{\dfrac{7}{24}x^3(\ln x)^3+o(x^3(\ln x)^3)}{-\dfrac{8}{3}x^3(\ln x)^3+o(x^3(\ln x)^3)} = -\frac{7}{64}, \end{aligned}\]

concludendo l’esercizio.


 
 

Esercizio 15. Sia f(x)=\sin^2(x+x^3).

    \[\quad\]

  1. Trovare lo sviluppo di Taylor di f di ordine 6 nel punto x=0.
  2.  

  3. Usare il risultato del punto (1) per calcolare f^{(6)}(0).
  4.  

  5. Calcolare g^{(2015)}(0) dove g(x)=x^2f(x).

Soluzione.

Ricordiamo che lo sviluppo del seno per x\to0 con resto di Peano è dato da

(43)   \begin{equation*} \sin x = \sum_{k=0}^n\frac{(-1)^k\,x^{2k+1}}{(2k+1)!}+o(x^{2n+1}), \end{equation*}

perciò per calcolare f(x), applichiamo questo sviluppo a \sin^2(x+x^3):

    \[\begin{aligned} \sin^2(x+x^3) &= \left(\sum_{k=0}^2\frac{(-1)^k(x+x^3)^{2k+1}}{(2k+1)!}+o(x^6)\right)^2 \\ &= \left(x+x^3-\frac{1}{3!}(x^3+x^5)+\frac{1}{5!}x^5+o(x^5)\right)^2 \\ &= \left(x+\frac{5}{6}x^3-\frac{59}{120}x^5+o(x^5)\right)^2 \\ &= x^2+2x\cdot\left(\frac{5}{6}x^3\right)+\frac{25}{36}x^6+2x\cdot\left(-\frac{59}{120}x^5\right)+o(x^6) \\ &= x^2+\frac{5}{3}x^4-\frac{13}{45}x^6+o(x^6). \end{aligned}\]

Dal teorema di Taylor (teorema 1) sappiamo che la derivata j-esima della funzione valutata in x_0 coincide con la derivata j-esima del polinomio di Taylor:

    \[ f^{(j)}(x_0) = P_n^{(j)}(x_0) \qquad\text{per ogni } j\in\mathbb{N}, \]

da cui segue che

    \[ f^{(6)}(0) = 6!\cdot\left(-\frac{13}{45}\right) = -208. \]

Infine, per calcolare g^{(2015)}(0), osserviamo che:

    \[\quad\]

  • f è una funzione pari:

        \[    f(-x) = \sin^2(-x-x^3) = (-\sin(x+x^3))^2 = \sin^2(x+x^3) = f(x).    \]

  •  

  • g(x) = x^2f(x) è il prodotto di due funzioni pari, dunque è pari
  •  

  • Lo sviluppo di Taylor di una funzione pari contiene solo termini di grado pari.

Di conseguenza, essendo 2015 dispari, concludiamo immediatamente che la derivata è necessariamente uguale a zero, ovvero g^{(2015)}(0) = 0.


 
 

Esercizio 16. Sia f(x)=(x-\arctan x)\ln(1+x^2+x^3)-\frac{x^5+x^6}{3}.

    \[\quad\]

  1. Calcolare, al variare di \alpha>0, il limite:

    (44)   \begin{equation*}    \lim_{x\to0^+}\frac{f(x)}{x^\alpha}    \end{equation*}

  2.  

  3. Usare il risultato del punto (1) per calcolare f^{(7)}(0).

Soluzione.

Sviluppiamo f(x) per x\to0 utilizzando gli sviluppi noti di arcotangente e logaritmo:

    \[\begin{aligned} f(x) &= \left(x-\left(x-\frac{x^3}{3}+\frac{x^5}{5}\right)+o(x^5)\right)\left(x^2+x^3-\frac{x^4}{2}+o(x^4)\right)-\frac{x^5}{3}-\frac{x^6}{3} \\ &= \left(\frac{x^3}{3}-\frac{x^5}{5}+o(x^5)\right)\left(x^2+x^3-\frac{x^4}{2}+o(x^4)\right)-\frac{x^5}{3}-\frac{x^6}{3} \\ &= \frac{x^5}{3}-\frac{x^7}{6}-\frac{x^7}{5}-\frac{x^5}{3}+o(x^7) \\ &= -\frac{11}{30}x^7+o(x^7). \end{aligned}\]

Sostituendo questo sviluppo in (44) otteniamo

    \[ \lim_{x\to0^+}\frac{f(x)}{x^\alpha} = -\frac{11}{30}\lim_{x\to0}x^{7-\alpha}(1+o(1)), \]

da cui segue il seguente comportamento al variare di \alpha:

    \begin{equation*} \lim_{x\to0^+}\frac{f(x)}{x^\alpha} = \begin{cases}    -\dfrac{11}{30} & \text{se }\alpha=7, \\[1em]    -\infty & \text{se }\alpha\in(0,7), \\[1em]    0 & \text{se }\alpha\in(7,+\infty). \end{cases} \end{equation*}

Dal teorema di Taylor, la derivata settima di f in zero coincide con il coefficiente del termine di grado 7 moltiplicato per il fattoriale 7!. In altre parole, si ha

    \[ f^{(7)}(0) = 7!\cdot\left(-\frac{11}{30}\right) = -1848. \]

 

Approfondimenti video sviluppi di Taylor

Leggi...

Di seguito sono presentati un video di teoria e cinque esercizi svolti in formato video sugli sviluppi di Taylor, a cura di Valerio Pattaro. Ecco i link al video di teoria e agli esercizi svolti:

 
 

Tutta la teoria di analisi matematica

Leggi...

  1. Teoria Insiemi
  2. Il metodo della diagonale di Cantor
  3. Logica elementare
  4. Densità dei numeri razionali nei numeri reali
  5. Insiemi Numerici \left(\mathbb{N},\, \mathbb{Z},\, \mathbb{Q}\right)
  6. Il principio di induzione
  7. Gli assiomi di Peano
  8. L’insieme dei numeri reali: costruzione e applicazioni
  9. Concetti Fondamentali della Retta Reale: Sintesi Teorica
  10. Costruzioni alternative di \mathbb{R}
  11. Binomio di Newton
  12. Spazi metrici, un’introduzione
  13. Disuguaglianza di Bernoulli
  14. Disuguaglianza triangolare
  15. Teoria sulle funzioni
  16. Funzioni elementari: algebriche, esponenziali e logaritmiche
  17. Funzioni elementari: trigonometriche e iperboliche
  18. Funzioni goniometriche: la guida essenziale
  19. Teorema di Bolzano-Weierstrass per le successioni
  20. Criterio del rapporto per le successioni
  21. Definizione e proprietà del numero di Nepero
  22. Limite di una successione monotona
  23. Successioni di Cauchy
  24. Il teorema ponte
  25. Teoria sui limiti
  26. Simboli di Landau
  27. Funzioni continue – Teoria
  28. Il teorema di Weierstrass
  29. Il teorema dei valori intermedi
  30. Il teorema della permanenza del segno
  31. Il teorema di Heine-Cantor
  32. Il teorema di esistenza degli zeri
  33. Il metodo di bisezione
  34. Teorema ponte versione per le funzioni continue
  35. Discontinuità di funzioni monotone
  36. Continuità della funzione inversa
  37. Teorema delle contrazioni o Teorema di punto fisso di Banach-Caccioppoli
  38. Teoria sulle derivate
  39. Calcolo delle derivate: la guida pratica
  40. Teoria sulle funzioni convesse
  41. Il teorema di Darboux
  42. I teoremi di de l’Hôpital
  43. Teorema di Fermat
  44. Teoremi di Rolle e Lagrange
  45. Il teorema di Cauchy
  46. Espansione di Taylor: teoria, esempi e applicazioni pratiche
  47. Polinomi di Taylor nei limiti: istruzioni per l’uso
  48. Integrali definiti e indefiniti
  49. Teorema fondamentale del calcolo integrale (approfondimento)
  50. Integrali ricorsivi
  51. Formule del trapezio, rettangolo e Cavalieri-Simpson
  52. Teoria sugli integrali impropri
  53. Funzioni integrali – Teoria
  54. Introduzione ai numeri complessi – Volume 1 (per un corso di ingegneria — versione semplificata)
  55. Introduzione ai numeri complessi – Volume 1 (per un corso di matematica o fisica)
  56. Serie numeriche: la guida completa
  57. Successioni di funzioni – Teoria
  58. Teoremi sulle successioni di funzioni
    1. 58a. Criterio di Cauchy per la convergenza uniforme
    2. 58b. Limite uniforme di funzioni continue
    3. 58c. Passaggio al limite sotto il segno di integrale
    4. 58d. Limite uniforme di funzioni derivabili
    5. 58e. Piccolo teorema del Dini
    6. 58f. Procedura diagonale e teorema di Ascoli-Arzela
  59. Serie di funzioni – Teoria
  60. Serie di potenze – Teoria
  61. Serie di Fourier – Teoria e applicazioni
  62. Integrali multipli — Parte 1 (teoria)
  63. Integrali multipli — Parte 2 (teoria e esercizi misti)
  64. Regola della Catena — Teoria ed esempi
  65. Jacobiano associato al cambiamento di coordinate sferiche
  66. Guida ai Massimi e Minimi: Tecniche e Teoria nelle Funzioni Multivariabili
  67. Operatore di Laplace o Laplaciano
  68. Teoria equazioni differenziali
  69. Equazione di Eulero
  70. Teoria ed esercizi sulla funzione Gamma di Eulero
  71. Teoria ed esercizi sulla funzione Beta
  72. Approfondimento numeri complessi
  73. Diverse formulazioni dell’assioma di completezza
  74. Numeri di Delannoy centrali
  75. Esercizi avanzati analisi

 
 

Tutte le cartelle di Analisi Matematica

Leggi...

  1. Prerequisiti di Analisi
    1. Ripasso algebra biennio liceo
    2. Ripasso geometria analitica
    3. Ripasso goniometria e trigonometria
    4. Errori tipici da evitare
    5. Insiemi numerici N,Z,Q,R
    6. Funzioni elementari
    7. Logica elementare
    8. Insiemi
  2. Successioni
    1. Teoria sulle Successioni
    2. Estremo superiore e inferiore
    3. Limiti base
    4. Forme indeterminate
    5. Limiti notevoli
    6. Esercizi misti Successioni
    7. Successioni per ricorrenza
  3. Funzioni
    1. Teoria sulle funzioni
    2. Verifica del limite in funzioni
    3. Limite base in funzioni
    4. Forme indeterminate in funzioni
    5. Limiti notevoli in funzioni
    6. Calcolo asintoti
    7. Studio di funzione senza derivate
    8. Dominio di una funzione
    9. Esercizi misti Funzioni
    10. Esercizi misti sui Limiti
  4. Funzioni continue-lipschitziane-holderiane
    1. Teoria sulle Funzioni continue-lipschitziane-holderiane
    2. Continuità delle funzioni
    3. Continuità uniforme
    4. Teorema degli zeri
    5. Esercizi sul teorema di Weierstrass senza l’uso delle derivate
  5. Calcolo differenziale
    1. Derivate
    2. Calcolo delle derivate
    3. Retta tangente nel calcolo differenziale
    4. Punti di non derivabilità nel calcolo differenziale
    5. Esercizi sul teorema di Weierstrass con l’uso delle derivate
    6. Studio di funzione completo nel calcolo differenziale
    7. Esercizi teorici nel calcolo differenziale
    8. Metodo di bisezione
    9. Metodo di Newton
  6. Teoremi del calcolo differenziale
    1. Teoria sui Teoremi del calcolo differenziale
    2. Teorema di Rolle
    3. Teorema di Lagrange
    4. Teorema di Cauchy
    5. Teorema di De L’Hôpital
  7. Calcolo integrale
    1. Integrale di Riemann
    2. Integrali immediati
    3. Integrale di funzione composta
    4. Integrali per sostituzione
    5. Integrali per parti
    6. Integrali di funzione razionale
    7. Calcolo delle aree
    8. Metodo dei rettangoli e dei trapezi
    9. Esercizi Misti Integrali Indefiniti
    10. Esercizi Misti Integrali Definiti
  8. Integrali impropri
    1. Teoria Integrali impropri
    2. Carattere di un integrale improprio
    3. Calcolo di un integrale improprio
  9. Espansione di Taylor
    1. Teoria Espansione di Taylor
    2. Limiti di funzione con Taylor
    3. Limiti di successione con Taylor
    4. Stime del resto
  10. Funzioni integrali (Approfondimento)
    1. Teoria Funzioni integrali (Approfondimento)
    2. Studio di funzione integrale
    3. Limiti con Taylor e De L’Hôpital
    4. Derivazione di integrali parametrici (Tecnica di Feynmann)
  11. Numeri Complessi
    1. Teoria Numeri complessi
    2. Espressioni con i numeri complessi
    3. Radice di un numero complesso
    4. Equazioni con i numeri complessi
    5. Disequazioni con i numeri complessi
    6. Esercizi misti Numeri complessi
  12. Serie numeriche
    1. Teoria Serie numeriche
    2. Esercizi Serie a termini positivi
    3. Esercizi Serie a termini di segno variabile
    4. Esercizi Serie geometriche e telescopiche
  13. Successioni di funzioni
    1. Teoria Successioni di funzioni
    2. Esercizi Successioni di funzioni
  14. Serie di funzioni
    1. Teoria Serie di funzioni
    2. Esercizi Serie di funzioni
  15. Serie di potenze
    1. Teoria Serie di potenze
    2. Esercizi Serie di potenze
  16. Serie di Fourier
    1. Teoria Serie di Fourier
    2. Esercizi Serie di Fourier
  17. Trasformata di Fourier
    1. Teoria Trasformata di Fourier
    2. Esercizi Trasformata di Fourier
  18. Funzioni di più variabili
    1. Teoria Funzioni di più variabili
    2. Massimi e minimi liberi e vincolati
    3. Limiti in due variabili
    4. Integrali doppi
    5. Integrali tripli
    6. Integrali di linea di prima specie
    7. Integrali di linea di seconda specie
    8. Forme differenziali e campi vettoriali
    9. Teorema di Gauss-Green
    10. Integrali di superficie
    11. Flusso di un campo vettoriale
    12. Teorema di Stokes
    13. Teorema della divergenza
    14. Campi solenoidali
    15. Teorema del Dini
  19. Equazioni differenziali lineari e non lineari
    1. Teoria equazioni differenziali lineari e non lineari
    2. Equazioni differenziali lineari e non lineari del primo ordine omogenee
  20. Equazioni differenziali lineari
    1. Del primo ordine non omogenee
    2. Di ordine superiore al primo,a coefficienti costanti,omogenee
    3. Di ordine superiore al primo,a coefficienti costanti,non omogenee
    4. Di Eulero,di Bernoulli,di Clairaut,di Lagrange e di Abel
    5. Non omogenee avente per omogenea associata un’equazione di Eulero
    6. Sistemi di EDO
  21. Equazioni differenziali non lineari
    1. A variabili separabiliO
    2. A secondo membro omogeneo
    3. Del tipo y’=y(ax+by+c)
    4. Del tipo y’=y(ax+by+c)/(a’x+b’y+c’)
    5. Equazioni differenziali esatte
    6. Mancanti delle variabili x e y
    7. Cenni sullo studio di un’assegnata equazione differenziale non lineare
    8. Di Riccati
    9. Cambi di variabile: simmetrie di Lie
  22. Analisi complessa
    1. Fondamenti
    2. Funzioni olomorfe
    3. Integrale di Cauchy e applicazioni
    4. Teorema della curva di Jordan e teorema fondamentale dell’Algebra
    5. Teorema di inversione di Lagrange
    6. Teorema dei Residui
    7. Funzioni meromorfe
    8. Prodotti infiniti e prodotti di Weierstrass
    9. Continuazione analitica e topologia
    10. Teoremi di rigidità di funzioni olomorfe
    11. Trasformata di Mellin
  23. Equazioni alle derivate parziali
    1. Equazioni del primo ordine
    2. Equazioni del secondo ordine lineari
    3. Equazioni non-lineari
    4. Sistemi di PDE
  24. Funzioni speciali
    1. Funzione Gamma di Eulero
    2. Funzioni Beta,Digamma,Trigamma
    3. Integrali ellittici
    4. Funzioni di Bessel
    5. Funzione zeta di Riemann e funzioni L di Dirichlet
    6. Funzione polilogaritmo
    7. Funzioni ipergeometriche
  25. Analisi funzionale
    1. Misura e integrale di Lebesgue
    2. Spazi Lp,teoremi di completezza e compattezza
    3. Spazi di Hilbert,serie e trasformata di Fourier
    4. Teoria e pratica dei polinomi ortogonali
    5. Spazi di Sobolev
  26. Complementi
    1. Curiosità e approfondimenti
    2. Compiti di analisi
    3. Esercizi avanzati analisi
  27. Funzioni Convesse

 
 

Tutti gli esercizi di geometria

In questa sezione vengono raccolti molti altri esercizi che coprono tutti gli argomenti di geometria proposti all’interno del sito con lo scopo di offrire al lettore la possibilità di approfondire e rinforzare le proprie competenze inerenti a tali argomenti.

Strutture algebriche.





 
 

Risorse didattiche aggiuntive per approfondire la matematica

Leggi...

  • Math Stack Exchange – Parte della rete Stack Exchange, questo sito è un forum di domande e risposte specificamente dedicato alla matematica. È una delle piattaforme più popolari per discutere e risolvere problemi matematici di vario livello, dall’elementare all’avanzato.
  • Art of Problem Solving (AoPS) – Questo sito è molto noto tra gli studenti di matematica di livello avanzato e i partecipanti a competizioni matematiche. Offre forum, corsi online, e risorse educative su una vasta gamma di argomenti.
  • MathOverflow – Questo sito è destinato a matematici professionisti e ricercatori. È una piattaforma per domande di ricerca avanzata in matematica. È strettamente legato a Math Stack Exchange ma è orientato a un pubblico con una formazione più avanzata.
  • PlanetMath – Una comunità collaborativa di matematici che crea e cura articoli enciclopedici e altre risorse di matematica. È simile a Wikipedia, ma focalizzata esclusivamente sulla matematica.
  • Wolfram MathWorld – Una delle risorse online più complete per la matematica. Contiene migliaia di articoli su argomenti di matematica, creati e curati da esperti. Sebbene non sia un forum, è una risorsa eccellente per la teoria matematica.
  • The Math Forum – Un sito storico che offre un’ampia gamma di risorse, inclusi forum di discussione, articoli e risorse educative. Sebbene alcune parti del sito siano state integrate con altri servizi, come NCTM, rimane una risorsa preziosa per la comunità educativa.
  • Stack Overflow (sezione matematica) – Sebbene Stack Overflow sia principalmente noto per la programmazione, ci sono anche discussioni rilevanti di matematica applicata, specialmente nel contesto della scienza dei dati, statistica, e algoritmi.
  • Reddit (r/Math) – Un subreddit popolare dove si possono trovare discussioni su una vasta gamma di argomenti matematici. È meno formale rispetto ai siti di domande e risposte come Math Stack Exchange, ma ha una comunità attiva e molte discussioni interessanti.
  • Brilliant.org – Offre corsi interattivi e problemi di matematica e scienza. È particolarmente utile per chi vuole allenare le proprie capacità di problem solving in matematica.
  • Khan Academy – Una risorsa educativa globale con lezioni video, esercizi interattivi e articoli su una vasta gamma di argomenti di matematica, dalla scuola elementare all’università.






Document